Female reproductive system Flashcards

1
Q

@# 6. Regarding yolk sac tumours of ovary:

A. Are the most common malignant germ cell tumour of the ovary

B. Account for 5% of all ovarian malignancy

C. Carry a poorer prognosis than any other ovarian germ cell tumour

D. Haemorrhagic change is very rare

E. Are slow growing tumours

A

C. Carry a poorer prognosis than any other ovarian germ cell tumour

Yolk sac tumours are well-enhanced tumours consisting of mixed solid and cystic tissue with some area of haemorrhage.

A ‘bright dot’ sign is recognised; a well-enhanced dilated vessel on the post-contrast image.

Yolk sac tumours have a poor prognosis.

They account for 1% of ovarian malignancies.

They are the second most common malignant germ cell tumour after dysgerminomas.

How well did you know this?
1
Not at all
2
3
4
5
Perfectly
2
Q
  1. Granulosa cell ovarian tumour is diagnosed following removal of a complex pelvic mass. Which is the single best answer?

A. Account for 15% of ovarian tumours

B. The juvenile subtype is more common

C. Has a rapid rate of growth

D. Recurrent disease is almost always in the frst two years after treatment

E. Variable imaging appearances are recognised from uniloculated cystic masses to solid masses

A

E. Variable imaging appearances are recognised from uniloculated cystic masses to solid masses

Represent 70% of malignant sex cord stromal tumours, but only 2-5% of all ovarian tumours have an unpredictable and indolent course with relapse occurring up to several years after initial diagnosis. The adult subtype accounts for 95% of all GCTs.

How well did you know this?
1
Not at all
2
3
4
5
Perfectly
3
Q
  1. Involvement of which of the following indicates the poorest prognosis in recurrent endometrial cancer?

A. Spleen

B. Vagina

C. Lung

D. Bladder

E. Well-differentiated tumour at original surgery

A

A. Spleen

Splenic, liver and multiple sites of disease are independent predictors of poor outcome.

How well did you know this?
1
Not at all
2
3
4
5
Perfectly
4
Q
  1. Regarding endometrial carcinoma on MR:

A. Normal zonal anatomy is best demonstrated on T1

B. Tumour is typically higher signal compared with endometrial lining on T2

C. Tumour is typically higher signal intensity than myometrium

D. Enhances faster than myometrium on dynamic contrast enhancement

E. Usually low SI than brightly enhancing normal myometrial tissue after contrast

A

E. Usually low SI than brightly enhancing normal myometrial tissue after contrast

Endometrial tumours are usually isointense to myometrium on T1 and lower SI to endometrial lining on T2. Tumours demonstrate slower enhancement on DCE than myometrium. Normal zonal anatomy is clear on T2.

How well did you know this?
1
Not at all
2
3
4
5
Perfectly
5
Q
  1. Which of the following ovarian masses appear more cystic than solid?

A. Arrhenoblastoma

B. Metastases

C. Fibroma

D. Lymphoma

E. Endometriosis

A

E. Endometriosis

Cystadenocarcinoma, dermoid abscess, endometriosis and ectopic pregnancy are examples of cystic ovarian masses.

How well did you know this?
1
Not at all
2
3
4
5
Perfectly
6
Q
  1. MRI shows two separate normal sized uteri and cervices with a septum extending into the upper vagina. The two uteri are widely separated, with preservation of the endometrial and myometrial widths. What name is given to this abnormality?

A. Uterus didelphys

B. Mullerian agenesis

C. Unicornate uterus

D. Uterus bicornuate

E. Septate uterus

A

A. Uterus didelphys

When partial fusion of the Müllerian ducts occurs, myometrium forms the dividing septum. This abnormality is known as a bicornuate uterus. A septate uterus arises when there is only partial resorption of the final fbrous septum dividing the two horns of the uterus. The latter defect results in further reproductive comp

How well did you know this?
1
Not at all
2
3
4
5
Perfectly
7
Q
  1. HSG shows small diverticular outpouchings in the isthmic portion of the right fallopian tube with distal tube occlusion. What is the diagnosis?

A. Tubal polyps

B. Salpingitis isthmica nodosa (SIN)

C. Adenomyosis

D. Asherman’s syndrome

E. Ectopic pregnancy

A

B. Salpingitis isthmica nodosa (SIN)

SIN is associated with pelvic inflammatory disease and a higher risk of ectopic pregnancy

How well did you know this?
1
Not at all
2
3
4
5
Perfectly
8
Q

@# 44. Regarding clear cell tumour of the ovary:

A. Are rarely invasive

B. Represents > 20% of ovarian carcinomas

C. Most patients present at stage 2 disease

D. Frequently occurs as a unilocular cyst with mural nodule

E. Has a poorer survival rate compared with other ovarian cancers

A

D. Frequently occurs as a unilocular cyst with mural nodule

50% of patients have a 5-year survival rate; it presents in stage I in 75% of cases and accounts for up to 10% of all ovarian cancers.

How well did you know this?
1
Not at all
2
3
4
5
Perfectly
9
Q
  1. Regarding mucinous ovarian tumours:

A. Are most commonly mucinous cystadenocarcinomas

B. Account for the most common benign epithelial neoplasias of the ovary

C. Are most common in the post-menopausal population

D. Rupture may lead to pseudomyxoma peritoneii

E. When mucinous, cystadenomas are unilocular cysts with few septa

A

D. Rupture may lead to pseudomyxoma peritoneii

20 % of ovarian tumours are mucinous.

These are the second most common benign epithelial neoplasm after serous ovarian neoplasias.

Mucinous cystadenomas account for 80% and are multiloculated cysts with numerous septae, occurring in the third to fifth decades.

How well did you know this?
1
Not at all
2
3
4
5
Perfectly
10
Q
  1. Regarding ovarian fbromas:

A. Demonstrate rapid enhancement on CECT

B. Commonly present as Meigs’ syndrome

C. Are usually bilateral

D. Usually hyperechoic on ultrasound

E. Low on T1 and T2, less or equal to myometrium

A

E. Low on T1 and T2, less or equal to myometrium

Well-defned solid masses in patients > 40.

Low SI on T1+T2, with poor delayed contrast enhancement.

Bilateral in 4-8% and associated with Meigs’ syndrome in 1%.

Meigs syndrome is defined as the triad of benign ovarian tumor with ascites and pleural effusion that resolves after resection of the tumor

How well did you know this?
1
Not at all
2
3
4
5
Perfectly
11
Q

3) A 30-year-old woman has a well-circumscribed, cystic, adnexal mass with areas of dense focal calcification, small enhancing soft-tissue elements, fluid–fluid levels and bright regions on T1W MRI that become dark on fat-saturated sequences. Which of the following pathologies is most likely?

a. ovarian cyst with proteinaceous contents

b. endometrioma

c. mature cystic teratoma of the ovary

d. ovarian cyst adenofibroma

e. ovarian adenocarcinoma

A

c. mature cystic teratoma of the ovary

The main differentials for an ovarian mature cystic teratoma (dermoid cyst) are endometriomas and proteinaceous ovarian cysts, which can also have fluid–fluid levels.

Fat is frequently demonstrated in a dermoid cyst, but not in these differentials.

Fat can be proven by a significant negative attenuation value on CT, or on MRI with chemical shift artefact in the frequency-encoding direction, a gradient echo sequence in which fat and water are in opposite phase or frequency-selective fat saturation sequences.

Mature cystic teratoma contains mature tissues of germ cell (pleuripotent) origin. At least two of the three germlines should be represented.

Mean patient age is 30 years, younger than for epithelial ovarian neoplasms, and it is the commonest ovarian mass in children.

Usually asymptomatic, they can cause abdominal pain or other nonspecific symptoms. They are bilateral in 10% of cases

How well did you know this?
1
Not at all
2
3
4
5
Perfectly
12
Q

8) A general practitioner performs a vaginal examination prior to intended removal of an intrauterine contraceptive device. The locator device cannot be seen or palpated. What is the most appropriate initial investigation for this patient?

a. abdominal radiograph

b. pelvic ultrasound scan

c. pelvic CT

d. pelvic MRI

e. hysteroscopy

A

b. pelvic ultrasound scan

The device should be seen within the endometrial cavity on ultrasound scan as an echo-bright structure casting an acoustic shadow. If it is not identified in the uterus on ultrasound scan, then a plain abdominal film is indicated to exclude perforation and migration.

How well did you know this?
1
Not at all
2
3
4
5
Perfectly
13
Q

12) A 40-year-old woman with a history of prior pelvic radiotherapy for cervical cancer has an ultrasound scan for cyclical pelvic pain. The endometrium is distended by predominantly echo-poor material, and both ovaries have moderately large cysts containing low-level echoes. On MRI, the cervix returns low T2 signal and the ovarian cysts return high signal on fat-suppressed T1W sequences. Which of the following is the most likely diagnosis?

a. recurrent cervical tumour with bilateral ovarian metastases

b. recurrent cervical tumour and synchronous bilateral ovarian teratomas

c. cervical stenosis and bilateral endometriomas

d. cervical stenosis and bilateral ovarian cystadenocarcinomas

e. new primary endometrial carcinoma with bilateral ovarian secondaries

A

c. cervical stenosis and bilateral endometriomas

Cervical stenosis can be congenital or acquired. When it is acquired, causes include cervical (after the menopause) or endometrial (before the menopause) carcinoma.

Radiation and curettage can also produce cervical stenosis.

On imaging, the endometrial cavity is distended by secretions and blood products.

Reflux endometriosis can complicate cervical stenosis

How well did you know this?
1
Not at all
2
3
4
5
Perfectly
14
Q

@# 13) A postmenopausal patient has a hysterectomy and bilateral salpingo-oophorectomy for bilateral ovarian masses. Histological examination confirms bilateral ovarian tumours and reveals concomitant endometrial adenocarcinoma. What is the most likely histological diagnosis of the ovarian lesions?

a. benign serous cystadenoma

b. benign mucinous cystadenoma

c. malignant serous cystadenocarcinoma

d. malignant mucinous cystadenocarcinoma

e. endometrioid tumour

A

e. endometrioid tumour

Benign serous cystadenoma is bilateral in 20% of cases, benign mucinous cystadenoma in 5%, malignant serous cystadenocarcinoma in 50% and malignant mucinous cystadenocarcinoma in 25%. However, not only are endometrioid ovarian tumours frequently bilateral (30–50%) but they are also often (30%) found with concomitant endometrial adenocarcinoma.

How well did you know this?
1
Not at all
2
3
4
5
Perfectly
15
Q

25) A 65-year-old female with biopsy-proven ovarian cancer has a staging CT scan. It reveals a left basal pleural effusion that after aspiration contains no malignant cytology. There is a large, complex, abdominopelvic mass, with ascites and peritoneal deposits outside the pelvis measuring over 2 cm in diameter. Pelvic and para-aortic lymph nodes are enlarged. There are liver surface and parenchymal deposits. Which of the described features results in a classification of stage IV disease?

a. ascites

b. pleural effusion

c. liver surface deposits

d. liver parenchymal deposits

e. 2 cm deposits outside the pelvis

A

d. liver parenchymal deposits

Liver capsule deposits are stage T3/III. The pleural effusion cannot be regarded as M1/IV, because it requires positive cytology for this. Any involved regional nodes give stage IIIc and include obturator, common, internal and external iliac, laterosacral, inguinal and para-aortic.

How well did you know this?
1
Not at all
2
3
4
5
Perfectly
16
Q

28) An imaging request is received with the clinical information, ‘biopsy-proven adenocarcinoma of the cervix, for local staging’. Which of the following is the most appropriate technique?

a. transvaginal ultrasound scan

b. endoanal ultrasound scan

c. CT abdomen and pelvis with intravenous and oral contrast

d. MRI with pelvic phased-array coil

e. 18FDG PET

A

d. MRI with pelvic phased-array coil

MRI is the technique of choice for local staging of cancer of the uterine cervix. CT is less useful for staging of the primary tumour but has value in detecting involved lymph nodes and distant metastases. 18FDG PET may be useful in some cases for detection of distant metastases or the identification of recurrent disease. Its value will vary with the histological diagnosis on account of varying radiotracer avidity, with squamous cell carcinomas typically being avid

How well did you know this?
1
Not at all
2
3
4
5
Perfectly
17
Q

32) A 23-year-old nulliparous woman is examined for dyspareunia. Biopsy confirms a clinically small but malignant-looking cervical lesion to be adenosquamous carcinoma. In such cases, local imaging staging must indicate which of the following?

a. tumour size and distance from the internal os plus the cervix length

b. tumour size and distance from the external os plus the uterine length

c. tumour size and distance from the vaginal introitus plus length of the vagina

d. tumour size and vascularity

e. ovarian position

A

a. tumour size and distance from the internal os plus the cervix length

Trachelectomy may be considered to conserve the uterus and preserve fertility in young women with small tumours.

Tumour size, distance from the internal os, cervix length and size of the uterus are required from the imaging.

Surgery, radiation and chemotherapy are treatment options for cervical cancer dependent on stage.

From 85% to 90% of cervical carcinomas have squamous cell histology, the remainder being mostly adenocarcinoma or adenosquamous.

How well did you know this?
1
Not at all
2
3
4
5
Perfectly
18
Q

33) A patient has a squamous cell carcinoma of the vulva. An MRI is performed for locoregional staging. There are significantly enlarged inguinal lymph nodes ipsilateral to the primary tumour, but none contralaterally. A short axis, ipsilateral, 1.2 cm external iliac node is also identified that has signal characteristics identical to the primary tumour throughout. Which of the following is the most accurate nodal staging?

a. Nx

b. N0

c. N1

d. N2

e. N3

A

c. N1

NX is used when regional nodes cannot be assessed, and N0 when there are no involved regional nodes.

N1 denotes ipsilateral involved femoral or inguinal lymph nodes.

N2 signifies bilateral regional nodal involvement.

All intrapelvic nodes are regarded as metastases and therefore do not influence the N stage.

There is no N3 for vulval cancer

How well did you know this?
1
Not at all
2
3
4
5
Perfectly
19
Q

35) A patient with endometrial cancer previously treated with surgery has an 18FDG PET scan to look for recurrence. A false-negative result could be caused by which of the following scenarios?

a. peritoneal deposits smaller than 1 cm

b. bladder diverticulum

c. post-surgical inflammation

d. abscess

e. bowel avidity

A

a. peritoneal deposits smaller than 1 cm

False positives can occur with PET because 18FDG is a metabolic tracer, and activity is seen in normal bowel, ovaries (cyclical), endometrium (cyclical), blood vessels, bone marrow and skeletal muscle. 18FDG is renally excreted; hence, focal accumulation can be seen in ureters, bladder diverticula, pelvic kidneys and urinary diversions. Benign processes can also take up this tracer, including abscesses, uterine fibroids, endometriosis, post-surgical inflammation, post-radiotherapy inflammation and sacral fractures. Fusion of the PETwith a CT scan can reduce these common pitfalls. However, using CT for attenuation correction can introduce other artefacts, such as apparently increased activity around metal prostheses. The PET acquisition is considerably longer than the CT one, allowing movement of bowel or bladder wall (with distension over time) and hence misregistration of PETactivity on the anatomical CT data. False-negative PET scans can be caused by small tumour deposits close to the urinary bladder, where they cannot be resolved from each other

How well did you know this?
1
Not at all
2
3
4
5
Perfectly
20
Q

43) A 45-year-old female has imaging to stage a cervical carcinoma. The primary tumour is 5 cm in longest dimension, is seen to involve the uterine corpus, and has small-volume parametrial spread that does not reach the pelvic side wall. Parametrial lymph nodes are significantly enlarged. There is no hydronephrosis. Vaginal involvement is also seen, with the caudal extent of the tumour being below the level of the urethral orifice into the bladder base. Which of the described features causes the local stage to be T3a?

a. size over 4 cm

b. uterine corpus invasion

c. parametrial spread

d. vaginal invasion

e. parametrial nodal involvement

A

d. vaginal invasion

The urethra is used as a landmark for the lower third of the vagina.

Cervical cancer involvement of the upper two-thirds of the vagina is T2a.

When the lower third is involved, it becomes T3a.

T3b disease denotes disease that reaches the pelvic side wall or has caused hydronephrosis.

Extension of disease into bladder or rectal mucosa is T4, as is disease extending out of the true pelvis.

Extension into the corpus only is disregarded.

T1b1 disease and T1b2 disease differ in being less or greater than 4cm respectively.

Parametrial lymph nodes are regional nodes and represent N1 disease; they do not influence the T stage

How well did you know this?
1
Not at all
2
3
4
5
Perfectly
21
Q

@# 47) A postmenopausal patient is investigated for ascites. Cytology from the ascites reveals cells in keeping with an epithelial ovarian malignancy. Which of the following is the most appropriate staging investigation?

a. CT of the abdomen and pelvis with oral and intravenous contrast

b. CT of the chest, abdomen and pelvis with oral and intravenous contrast

c. MRI of the pelvis

d. 18FDG PET

e. PET/CT

A

a. CT of the abdomen and pelvis with oral and intravenous contrast

Plain chest radiograph may be added to this as a routine, but chest CT would be requested only with an additional reason to do so. MRI of the ovaries can be helpful in characterizing ovarian masses where ultrasound scan and CA-125 are equivocal. There may be a role for PET/CT in defining disease extent, but cystic tumour deposits, particularly when they may be on or close to bowel or associated with ascites, present a challenge for this technique.

How well did you know this?
1
Not at all
2
3
4
5
Perfectly
22
Q

55) Lymphatic drainage from the lower third of the vagina is normally first to which of the following LN groups?
a. obturator

b. internal iliac

c. external iliac

d. inguinal

e. retroperitoneal

A

d. inguinal

The upper two-thirds of the vagina drain to the pelvic nodes, which is of relevance when imaging vaginal cancer. This cancer is uncommon, representing 1–2% of gynaecological malignancy. Eighty-five per cent of cases of vaginal cancer are squamous and 15% are adenocarcinoma. Clear-cell carcinoma is a rare form of adenocarcinoma found in young patients with in utero diethylstilbestrol exposure. Even less common are melanoma, sarcoma and adenosquamous carcinoma occurring as vaginal primaries. The two commonest cell types have different natural histories. Adenocarcinoma tends to involve pelvic and is more likely to involve supraclavicular lymph nodes, while squamous carcinomas are more likely to give rise to liver metastases. They are equally likely to metastasize to the lungs.

How well did you know this?
1
Not at all
2
3
4
5
Perfectly
23
Q

59) A 25-year-old female undergoes ultrasound scan of the pelvis for low abdominal pain. A gas reflection is seen within the uterine cavity. Which of the following is the likely cause of the pain?

a. endometriosis

b. adenomyosis

c. endometritis

d. endometrial carcinoma

e. tubo-ovarian abscess

A

c. endometritis

Endometritis is the commonest cause of gas in the uterus. Gas is also seen in the uterus when a submucosal fibroid becomes infected, when necrotic neoplastic tissue is metabolized by bacteria, because of fistula to the gastrointestinal tract, in pyometra secondary to cervix obstruction by cancer, or in cases of gas gangrene due to clostridial infection following septic abortion. Ovarian gas can be seen with infection within an ovarian neoplasm. Numerous gas-filled spaces in the vaginal submucosa and exocervix can occur in pregnancy; this is termed ‘vaginitis emphysematosa’.

How well did you know this?
1
Not at all
2
3
4
5
Perfectly
24
Q

@# 60) MRI is performed for locoregional staging of vaginal cancer. Which of the following descriptions is the most likely appearance on a T2W sequence, given a small primary tumour confined to the vagina?

a. central high signal within the vagina; focal homogeneous, low-signal mass not breaching the surrounding ring of intermediate-signal vaginal wall

b. central high signal within the vagina; focal homogeneous, high-signal mass not breaching the surrounding low-signal vaginal wall

c. central high signal within the vagina; focal homogeneous, intermediate-signal mass breaching the surrounding low-signal vaginal wall

d. central high signal within the vagina; focal homogeneous, intermediate-signal mass not breaching the surrounding low-signal vaginal wall

e. central intermediate signal; focal homogeneous, high-signal mass contained by low-signal vaginal wall

A

d. central high signal within the vagina; focal homogeneous, intermediate-signal mass not breaching the surrounding low-signal vaginal wall

The vaginal epithelial layer and mucus are bright on T2W images. This is normally surrounded by low-signal (fibromuscular) vaginal wall. Tumours are typically intermediate signal on T2W images. If gadolinium is used, cancers often have early phase enhancement. Large tumours may have central necrosis.

T1 tumours do not breach the low-T2-signal vaginal wall,

whereas T2 tumours do and extend into the paracolpal fat.

T3 tumours reach the pelvic side wall

while T4 tumours extend beyond the true pelvis or involve bladder or rectal mucosa

How well did you know this?
1
Not at all
2
3
4
5
Perfectly
25
Q

61) A 30-year-old, nulliparous woman with Stein–Leventhal syndrome is being treated for subfertility with clomiphene. She develops abdominal pain, distension, nausea and vomiting. Ultrasound examination of the abdomen reveals both ovaries to be larger than 7 cm in length and packed with large follicles, and also reveals an ovarian cyst 12 cm in diameter. Ascites and a pleural effusion are also seen. What is the most likely diagnosis?

a. endometriosis

b. ovarian cyst torsion

c. ovarian hyperstimulation syndrome

d. ovarian serous cystadenoma

e. corpus luteum of menstruation

A

c. ovarian hyperstimulation syndrome

Ovarian hyperstimulation syndrome is more commonly seen with human menopausal gonadotrophin therapy but can also be seen with clomiphene. Severe complications relate to volume depletion, such as hypovolaemia, oliguria, electrolyte imbalance and thromboembolic events. Intra-abdominal haemorrhage is also reported.

How well did you know this?
1
Not at all
2
3
4
5
Perfectly
26
Q

@# 63) On transvaginal ultrasound scan, an ovary measures 5 X 3 X 2 cm. Regarding the volume of this ovary, which of the following statements is most accurate?

a. it is large for pre- and postmenopausal ovaries

b. it is normal for pre- and postmenopausal ovaries

c. it is normal for a premenopausal ovary but large for a postmenopausal ovary

d. it is normal for a postmenopausal ovary but large for a premenopausal ovary

e. not enough information is given to assess the volume

A

c. it is normal for a premenopausal ovary but large for a postmenopausal ovary

Normal ovarian volume is less than 18 cm3 before the menopause and less than 8 cm3 after.

The volume can be estimated by multiplying the three diameters and dividing by two.

How well did you know this?
1
Not at all
2
3
4
5
Perfectly
27
Q

65) A transvaginal ultrasound scan is performed on a premenopausal woman on day 21 of the menstrual cycle. Given that her endometrium is normal, which of the following measurements of endometrial thickness is most likely?
a. 2 mm

b. 2–4 mm

c. 4–8 mm

d. 7–14 mm

e. greater than 14 mm

A

d. 7–14 mm

The menstrual endometrium is under 4mm.

After menstruation and up to date 14, the proliferative endometrium is 4–8mm.

Days 14–28 are secretory with the endometrium 7–14mm.

On ultrasound scan, the endometrium is seen as an echo-bright stripe. Unless the patient is taking tamoxifen or hormones, the postmenopausal endometrium should be less than 4mm.

A cut-off of 3mm when performing screening for endometrial cancer has a 99% negative predictive value.

How well did you know this?
1
Not at all
2
3
4
5
Perfectly
28
Q

69) A 68-year-old female patient has a pelvic ultrasound scan for a palpable mass. Arising within the left ovary is a 15 cm cyst with an irregular thick wall, frond-like solid elements, multiple septations over 2 mm thick and a pulsatility index of 0.5. These sonographic appearances are most in keeping with which of the following ovarian cystic structures?

a. corpus luteum cyst

b. follicular cyst

c. polycystic ovaries

d. benign ovarian neoplasm

e. malignant ovarian neoplasm

A

e. malignant ovarian neoplasm

Features of an ovarian cyst that suggest malignancy are thick irregular walls and thick septations (.2mm), large overall size, solid elements and, on Doppler scan, a high peak systolic velocity and low-impedance diastolic flow. Together, these give a resistive index (RI) of ,0.4 and a pulsatility index (PI) of ,1.

How well did you know this?
1
Not at all
2
3
4
5
Perfectly
29
Q

@# 70) A 17-year-old female with primary amenorrhoea is found on clinical examination to have a hypoplastic upper/middle vagina. MRI shows an absent uterus but normal tubes and ovaries. Which of the following is the most likely diagnosis?

a. uterus didelphys

b. unicornuate uterus

c. Mayer–Rokitansky–Kuster–Hauser syndrome

d. uterine agenesis

e. septate uterus

A

c. Mayer–Rokitansky–Kuster–Hauser syndrome

The uterus, fallopian tubes and upper vagina arise from the paired paramesonephric (mullerian) ducts. The caudal parts fuse and ultimately form the uterus and upper vagina with resorption of the midline septum. The cranial parts remain unfused and form the fallopian tubes. Congenital uterine abnormalities arise with failure of development or fusion of this duct, or failure of midline resorption following fusion. Mayer–Rokitansky–Ku¨ster–Hauser syndrome describes uterine agenesis accompanied by hypoplastic proximal/middle third of the vagina but normal tubes and ovaries. Forty per cent of patients with the syndrome have pelvic kidneys and other urinary tract anomalies are also associated. They have a normal genotype.

How well did you know this?
1
Not at all
2
3
4
5
Perfectly
30
Q

72) A postmenopausal woman is found on MRI to have a multicystic adnexal mass that contains fluid–fluid levels and does not show any fat suppression. In addition, her uterus shows a widened junctional zone containing small bright foci on T2W images. For which of the following diseases is she most likely to be receiving oral treatment that can account for these findings?

a. urinary tract infection

b. deep venous thrombosis

c. endometrial cancer

d. breast cancer

e. bipolar disorder

A

d. breast cancer

The patient is receiving tamoxifen. Side effects include subendometrial cystic atrophy, endometrial hyperplasia and endometrial polyps. Less frequent side effects are endometriosis, polypoid endometriosis, adenomyosis and cervical polyps. There is an increased risk of endometrial carcinoma. On MRI, an endometrioma can appear as a multicystic adnexal mass of high T1 and both hypo- and hyperintense T2 signal, but without the fat suppression that would be expected with a mature cystic teratoma. Adenomyosis on MRI may manifest as a uterus with a thickened, low-signal, junctional zone on T2W images, containing small foci of high T2 signal.

How well did you know this?
1
Not at all
2
3
4
5
Perfectly
31
Q

73) An 80-year-old female is found incidentally to have a unilateral, unilocular, echo-free, thin-walled ovarian cyst of diameter 4 cm. There are no papillary projections or solid parts, and the CA-125 is less than 30 U/ml. Which of the following is the most appropriate management?

a. pelvic exenteration

b. total abdominal hysterectomy and bilateral salpingo-oophorectomy

c. bilateral oophorectomy

d. laparoscopic staging

e. repeat transvaginal ultrasound scan in 4 months

A

e. repeat transvaginal ultrasound scan in 4 months

The Risk of Malignancy Index (RMI) is used to stratify the likelihood of an incidentally identified ovarian cyst being malignant. The RMI is the product of the ultrasound score, the CA-125 level and the score assigned according to menopausal status. Low-risk cysts can be managed conservatively. A cyst below 5cm in diameter that is unilocular, unilateral and echo-free, and has no solid parts or papillary formations has a risk of malignancy of less than 1%, and a 50% chance of resolving spontaneously in 3 months.

How well did you know this?
1
Not at all
2
3
4
5
Perfectly
32
Q

(Ped) 73) An incidental finding made in a 13-year-old girl is of unilateral ovarian atrophy. The atrophic ovary has stippled calcification. Given these features, which is the most likely explanation?

a. ovarian teratoma

b. amputated ovary

c. follicular ovarian cyst

d. ovarian leiomyoma

e. ovarian vein thrombosis

A

b. amputated ovary

An amputated ovary occurs as the result of ovarian torsion and infarction. Both an ovarian teratoma and leiomyoma will enlarge the affected ovary rather than appear atrophic. A follicular cyst will usually be a simple cyst, although it may have internal echoes produced by haemorrhage.

How well did you know this?
1
Not at all
2
3
4
5
Perfectly
33
Q

76) A 70-year-old woman is known to have uterine fibroids. There has been a clinically apparent increase in the uterine size. Transvaginal ultrasound appearances are in keeping with a large myometrial fibroid. Which of the following diagnoses must be considered in this patient?

a. lipoleiomyoma

b. endometrial hyperplasia

c. adenomyoma

d. leiomyosarcoma

e. Bartholin’s gland tumour

A

d. leiomyosarcoma

Uterine fibroids are oestrogen dependent and should involute following the menopause. Increase in size of a fibroid after the menopause should raise the possibility of sarcomatous degeneration. On ultrasound scan, the appearance of leiomyosarcoma may be indistinguishable from that of a benign fibroid

How well did you know this?
1
Not at all
2
3
4
5
Perfectly
34
Q

78) A 28-year-old female has a hysterosalpingogram for infertility. Both fallopian tubes distend progressively with contrast injection but without peritoneal spill of contrast. A delayed plain abdominal radiograph shows continued distension of both tubes by dense collections of contrast and no peritoneal spill. Given these findings, which of the following is the most likely predisposition to infertility for this patient?

a. tuberculosis

b. endometriosis

c. pelvic inflammatory disease

d. submucosal uterine fibroids

e. Asherman’s syndrome

A

c. pelvic inflammatory disease

The patient’s fallopian tubes are occluded, giving bilateral hydrosalpinx. The commonest cause of proximal or distal tubal occlusion is pelvic inflammatory disease. Endometriosis, infection following birth or abortion, and tuberculosis are other causes. Indications for hysterosalpingogram include infertility, recurrent miscarriage, assessment of the tubes after surgery and assessment of the integrity of a post-caesarean uterine scar. Contraindications are pregnancy, purulent vulval or cervical discharge, pelvic inflammatory disease in the preceding 6 months and contrast sensitivity. Historical contraindications include the immediate post-menstruation phase and recent dilatation and curettage, because of the risks associated with intravasation of oily contrast media.

How well did you know this?
1
Not at all
2
3
4
5
Perfectly
35
Q

83) A 28-year-old female is investigated for infertility. She has raised androgen levels and a higher than normal luteinizing hormone: follicle-stimulating hormone ratio. Pelvic ultrasound scan demonstrates bilaterally large ovaries with multiplesmall follicles. Which of the following is the most likely reason for the patient’s infertility?

a. cervical fibroids

b. hostile cervical mucus

c. ovarian torsion

d. polycystic ovarian disease

e. bilateral ovarian endometriosis implants

A

d. polycystic ovarian disease

Polycystic ovarian disease is diagnosed by clinical, biochemical and ultrasound findings. Clinically, oligomenorrhoea, hirsutism and obesity are features. Luteinizing hormone is increased as is the luteinizing hormone: follicle-stimulating hormone ratio. Androgen levels are increased. Sonographic findings vary from normal-looking ovaries through hypoechoic ovaries without individual cysts to multiple, 5mm or more, peripherally located cysts in bilaterally larg

How well did you know this?
1
Not at all
2
3
4
5
Perfectly
36
Q

86) A 79-year-old female has a 6-month history of vaginal bleeding. Transvaginal ultrasound scan demonstrates an ill-defined endometrium measuring 20 mm in thickness. Outpatient clinic endometrial biopsy confirms endometrial adenocarcinoma. MRI stage is T4. Which of the following MRI features supports this stage?

a. disease limited to the endometrium

b. cancer invasion evident into the outer half of the myometrium

c. vaginal involvement

d. rectal serosal involvement

e. bladder mucosal involvement

A

e. bladder mucosal involvement

Endometrial carcinoma is the commonest gynaecological malignancy, and the fourth commonest site of female malignancy. It is very unlikely if the endometrial thickness on transvaginal ultrasound scan is less than 4mm. Endometrial carcinoma becomes stage T4 when bladder or bowel mucosa is involved, whereas the stage remains T3 if other layers of bowel or bladder are invaded. On MRI, endometrial carcinoma has homogeneous signal intensity, isointense to myometrium on T1W images and hypointense to endometrial lining on T2W images. Endometr cancers shows slower contrast enhancement to a lower peak of enhancement than normal myomet.

How well did you know this?
1
Not at all
2
3
4
5
Perfectly
37
Q

89) On a midline sagittal T2W MR image of the uterus of a 25-year-old female, the endometrium, junctional zone and outer myometrium of the corpus are clearly identified. From innermost to outermost, which of the following signal intensities best describes the normal uterus?

a. high, intermediate, low

b. high, low, intermediate

c. intermediate, high, low

d. intermediate, low, high

e. low, intermediate, high

A

b. high, low, intermediate

The premenopausal uterus normally has a bright endometrium within a dark junctional zone and an intermediate outer myometrium on T2W images. Cancer disrupts the zonal anatomy seen on T2W MRI. T1W sequences do not demonstrate uterine zonal anatomy. On T2W images, the uterine cervix has a distinct zonal signal pattern that is particularly well seen on sequences acquired perpendicular to the long axis of the cervix. These are especially useful for cervical cancer staging. The cervix lumen is bright, the cervical mucosa is intermediate to bright, and the fibromuscular cervical stroma is dark and surrounded by the intermediate signal outer layer of cervical stroma. With age or radiation treatment, the uterus involutes and loses this zonal appearance on T2W MRI. Notably, the outer corpus and especially the cervical muscles return a lower signal.

How well did you know this?
1
Not at all
2
3
4
5
Perfectly
38
Q

90) A 60-year-old female with urinary retention and pelvic pain is investigated with MRI of the pelvis. On sagittal T2W images, the bladder is seen to be 2 cm below the pubococcygeal line. What is cause of patient’s symptoms?

a. anterior rectocele

b. enterocele

c. cystocele

d. rectal prolapse

e. bladder intussusception

A

c. cystocele

Suspected pelvic floor prolapse can be investigated with MRI. Usually, there is minimal movement of pelvic organs even on maximal strain. When the pelvic floor is lax, the organs descend below the pubococcygeal line by 1–2cm. When descent exceeds 2cm, the prolapse may require surgical intervention. An enterocele describes small bowel descending 2cm or more between vagina and rectum. Anterior bulging of the rectal wall is known as a rectocele, while bladder descent of more than 1cm is a cystocele.

How well did you know this?
1
Not at all
2
3
4
5
Perfectly
39
Q

91) Regarding normal pelvic floor anatomy, which of the following is contained within the middle compartment of the female pelvic floor?

a. bladder

b. urethra

c. vagina

d. rectum

e. anus

A

c. vagina

The pelvic floor is supported by the endopelvic fascia and the levator ani muscle complex. This complex consists of three muscle groups, iliococcygeal, pubococcygeal and puborectalis. The anterior compartment of the female pelvic floor contains the bladder and urethra. The middle compartment contains the vagina and the posterior compartment the rectum.

How well did you know this?
1
Not at all
2
3
4
5
Perfectly
40
Q

99) Which of the following is the strongest indication for a PET/CT scan?

a. cervical cancer staging

b. endometrial cancer staging

c. ovarian cancer staging

d. prostate cancer staging

e. bladder transitional cell carcinoma staging

A

a. cervical cancer staging

In the pelvis, 18FDG PET/CT is recommended for use in staging colorectal cancer, cervical cancer and non-Hodgkin’s lymphoma. It is also indicated for detecting recurrence of colorectal, cervical, endometrial and ovarian cancers. It is not recommended for primary urothelial malignancy or prostate cancer. Usefulness is limited in the renal tract by urinary excretion of FDG. It is less than satisfactory in prostate cancer because of the poor sensitivity for osseous metastases

How well did you know this?
1
Not at all
2
3
4
5
Perfectly
41
Q

9 A 26 year old, otherwise fit and well female patient is referred for a pelvic US as part of her routine investigations for infertility. US shows an 8 cm right complex adnexal mass with echogenic and anechoic components. CT shows a inass of fat density floating in an interface between two water density components. MRI shows a hyperintense mass on T2W lesionwith a fluid-fluid level. What is the likeliest diagnosis?

(a) Tubo-ovarian abscess

(b) Endometrioma

(c) Ovarian carcinoma

(d) Dermoid cyst

(e) Haemorrhagic cyst

A

(d) Dermoid cyst

Dermoid cysts are common congenital benign germ cell tumours that usually present in the reproductive age range. 15% are bilateral. They have a spectrum of appearances ranging from the classic fat or bone containing lesions to (less commonly) a predominantly cystic lesion with a fluid-fluid level. They can be diagnosed on plain radiographs in up to 40% of cases when the presence of fat, teeth or bone can be identified. They can be complicated by malignant degeneration (in 1-3%), torsion (4-16%) and rupture (rarely).

How well did you know this?
1
Not at all
2
3
4
5
Perfectly
42
Q

12 A 24 year old female patient with a regular 28 day menstrual cycle undergoes a transvaginal US examination as part of her investigations for dyspareunia. She cannot recall when her last menstrual period began. US shows bright central line, with a markedly echogenic smooth endometrium measuring 14 mm. Mild echogenic posterior acoustic enhancement is seen with a thin hypoechoic halo of inner myometrial zone. Which is correct?

(a) She should be referred for biopsy/dilation and curettage

(b) She is in the proliferative phase of her menstrual cycle

(c) She is in the secretory phase of her menstrual cycle

(d) The report should not mention her unknown LMP

(e) She is in the ovulatory phase of her menstrual cycle

A

(c) She is in the secretory phase of her menstrual cycle

An endometrial thickness of up to 16 mm may be seen in the secretory phase of the menstrual cycle. This is maximal during the mid-secretory phase. It is imperative to include the LMP, whether known or unknown as part of the report. In the proliferative phase (day 6-14), endometrial thickness is 5-7 mm and in the periovulatory phase, endometrial thickness up to 11 mm is seen.

How well did you know this?
1
Not at all
2
3
4
5
Perfectly
43
Q

14 A 45 year old woman undergoes a PET-CT as part of her investigations for cervical cancer. Which of the following is the least likely to cause a false positive result?

(a) Physiological uptake in bowel

(b) Bladder diverticulum

(c) Uterine fibroids

(d) Endometriosis

(e) Ovarian cyst

A

(e) Ovarian cyst

18FDG shows physiological uptake in brain, myocardium, liver, spleen, bone marrow, GI tract, testes, and skeletal muscle and is excreted by the kidneys. Increased uptake is also seen in healing fractures, inflammatory and granulomatous disease and infectious processes.

How well did you know this?
1
Not at all
2
3
4
5
Perfectly
44
Q

16 A 32-year-old woman with a history of multiple previous basal cell carcinomas of the skin undergoes a pelvic US. This shows a 5 cm solid-looking hypoechoic left ovarian mass. MRI shows the mass is well circumscribed, relatively homogeneous and of low signal intensity on T1 and T2. What is the most likely diagnosis?

(a) Ovarian adenocarcinoma

(b) Cystadenocarcinoma

(c) Brenner tumour

(d) Ovarian fibroma

(e) Endometrioma

A

(d) Ovarian fibroma

Ovarian fibroma is a well defined tumour containing extensive collagen. It is associated with Gorlin’s syndrome (fibromas seen in 17%) and occasionally presents as Meig’s syndrome.

How well did you know this?
1
Not at all
2
3
4
5
Perfectly
45
Q

17 A 19 year old female patient presents with a history of amenorrhoea. On examination she is noted to be overweight, hirsute, the blood sugar is raised on pin-prick testing. An underlying endocrine disorder is suspected. Which of the following features would not be in keeping with this diagnosis?

(a) Bilateral ovarian volumes of 20 m/s

(b) Bilateral ovarian volumes of 5 m/s

(c) 10 sman peripheral 5-8 mm cystic lesions on either side

(d) Asymmetrical ovarian size

(e) Patient’s age

A

(d) Asymmetrical ovarian size

The diagnosis of polycystic ovary syndrome is made on the basis of clinical and biochemical findings in conjunction with the sonographic findings, which alone are non-specific. Similar sized ovaries is a key finding. The classic finding of bilaterally enlarged ovaries with multiple small peripheral follicles is seen in 50%. Hyperechoic central stroma is also usually seen. Normal ovaries are seen in 25%.?

How well did you know this?
1
Not at all
2
3
4
5
Perfectly
46
Q

18 With regards to MRI protocols for the assessment of endometriosis, which of the following statements is incorrect?

(a) A pelvic surface coil improves image quality

(b) T1 W sequences post gadolinium administration is employed

(c) T2W spin echo sequences are routinely employed

(d) A smooth muscle relaxant improves image quality

(e) T2W gradient echo sequences are not routinely employed

A

(b) T1 W sequences post gadolinium administration is employed

T1W imaging after the administration of intravenous gadolinium is not routinely employed for the assessment of endometriosis.

How well did you know this?
1
Not at all
2
3
4
5
Perfectly
47
Q

28 A 34 year old woman presents with left lower abdominal pain. US shows a complex adnexal lesion which cannot be fully characterised. MRI shows a 5 cm fluid filled structure with incomplete non-enhancing plicae crossing it and with areas of focal narrowing and peripheral small low signal intensity nodules on T2W. Amorphous shading of the lesion is also seen on T2W. Enhancement of the wall of the lesion after gadolinium administration is noted. Normal ovarian tissue is seen stretched around the lesion. What is the likely diagnosis?

(a) Endometriosis

(b) Ovarian mucinous carcinoma

(c) Fallopian tube carcinoma

(d) Pyosalpinx

(e) Lymphangioma

A

(d) Pyosalpinx

Pyosalpinx is characterized by dilatation of the fallopian tube with pus within it. It characteristically demonstrates amorphous or geographic shading on T2W

How well did you know this?
1
Not at all
2
3
4
5
Perfectly
48
Q

37 A 37 year old female patient with suspected pelvic malignancy undergoes an MRI with diffusion weighted imaging. There is an area of slight T2 hyperintensity which shows high signal intensity on high b-value source images and decreased signal on the ADC map. What is the most likely diagnosis?

(a) Liquefactive necrosis

(b) Fibrous tissue

(c) T2.:.shine through

(d) Cyst

(e) High-cellularity tumour

A

(e) High-cellularity tumour

Diffusion-weighted MRI is increasingly being used in body MRI and offers functional information over and above conventional morphological images. These features are typical of tumour.

How well did you know this?
1
Not at all
2
3
4
5
Perfectly
49
Q

50 A 14 year old girl presents with lower abdominal pain. Transabdominal ultrasound shows a partially solid/partially cystic midline mass. Which of the following radiological features would not support a diagnosis of ovarian torsion?

(a) Enhancement of the solid component on CT

(b) Free fluid in the cul-de-sac on US

(c) Multiple peripheral cysts on US

(d) High signal intensity on fat suppressed T1

(e) Wall thickness of 12 mm on MRI

A

(a) Enhancement of the solid component on CT

Lack of enhancement of the solid component of the mass is seen on CT and MRI. Other features include a whorled paraovarian structure on CT, representing the ovarian pedicle

How well did you know this?
1
Not at all
2
3
4
5
Perfectly
50
Q

21 A 30 year old woman presents with chronic pelvic pain and dyspareunia. PV examination reveals tenderness in ‘the right adnexa. Pelvic US shows a 4 cm cystic lesion arising from the right ovary with diffuse homogeneous low-levet internal echoes and echogenic wall foci. What is the likeliest diagnosis?

(a) Haemorrhagic ovarian cyst

(b) Endometrioma

(c) Dermoid

(d) Cystadenocarcinoma

(e) Tubo-ovarian abscess

A

(b) Endometrioma

The classic features of an endometrioma are described, others include wall nodularity, a fluid-fluid level and acoustic enhancement. A haemorrhagic ovarian cyst or a tubo-ovarian abscess wou1d present more acutely. A dermoid may have similar appearances but would usually have fat and/or calcium within. A cystic ovarian tumour is in the differential and typically appears as a multilocular cyst with large associated soft tissue component mass, sometimes with papillary excrescences into the cysts.

How well did you know this?
1
Not at all
2
3
4
5
Perfectly
51
Q

29 A 28 year old woman is referred for investigations for infertility. HSG and MRI demonstrate normal fundal contour but no division of the uterine horns and a single uterine canal with a saddle shaped fundus. What is the most likely diagnosis?

(a) Arcuate uterus

(b) Septate uterus

(c) Bicornuate uterus

(d) Uterus d)delphys

(e) Unicornuate uterus

A

(a) Arcuate uterus

Arcuate uterus is the most common uterine anomaly associated with reproductive failure. Uterus didelphys is a duplication defect with 2 vaginas, 2 cervices and 2 uterine horns. Bicornuate and unicornuate abnormalities demonstrate an abnormal fundal contour.

How well did you know this?
1
Not at all
2
3
4
5
Perfectly
52
Q

31 A 73 year old woman is referred to the post-menopausal bleed fast track clinic where she undergoes a transvaginal US. Which of the following statements is incorrect?

(a) Endometrial measurement should be of both opposed endometrial layers

(b) The normal endometrial thickness measurement should be less than 5 mm

(c) If she is taking HRT, normal endometrial thickness can be up to 20 mm

(d) Normal measurement limits can be increased by 1-2 mm in obese patients

(e) Biopsy/D+C is advisable if she is not on HRT and has an endometrial thickness of >8 mm

A

(c) If she is taking HRT, normal endometrial thickness can be up to 20 mm

HRT can increase the normal thickness of the post-menopausal endometrium up to 15 mm (depending on the HRT). Measurements should be of bi-layer thickness. For post-menopausal women not on HRT, 5 mm is the upper limit of normal.

How well did you know this?
1
Not at all
2
3
4
5
Perfectly
53
Q

35 A 26 year old woman who has a Mirena IUCD in situ presents with intermittent lower abdominal pain. Which of the following statements is correct?

(a) 3T MRI is contraindicated

(b) Mirena IUCDs are better visualised than copper IUCDs on transabdominal US

(c) Threads of a lost coil are not visible on ultrasound

(d) IUCDs increase the risk of ectopic pregnancy

(e) The Mirarla is commonly used for the treatment of dysfunctional uterine bleeding

A

(e) The Mirarla is commonly used for the treatment of dysfunctional uterine bleeding

The Mirena is a polyethylene IUCD and is well seen on MRI. Metal IUCDs are not a contraindication to 1.5T MRI examination although safety in a 3T machine has not been fully evaluated. Threads of a lost coil can ball up to form an echogenic mass on ultrasound. IUCDs do not increase the risk of ectopic pregnancy per se, but a higher percentage of ectopic pregnancies are seen in patients with IUCDs as endometrial implantation is restricted. The Mirena is commonly used for treatment of dysfunctional uterine bleeding with success rates of up to 70%.

How well did you know this?
1
Not at all
2
3
4
5
Perfectly
54
Q

@#e 36 A 30 year old woman with a previous history of pelvic inflammatory disease is undergoing investigations for infertility. With regards to assessment of tubal patency, which of the following statements is correct?

(a) Gadolinium enhanced MRI is the investigation of choice

(b) Normal fallopian tubes are visible on pelvic US

(c) Hysterosalpingography should be performed in the first half of the menstrual cycle

(d) Hysterosalpingo contrast sonography is as good as laparoscopy and dye instillation

(e) Iodine based contrast is used for hysterosalpingo contrast sonography

A

(c) Hysterosalpingography should be performed in the first half of the menstrual cycle

Laparoscopy and blue dye instillation (with spillage of dye into the peritoneal cavity indicating patency) is the gold standard, but requires a general anaesthetic. MRI and standard pelvic US do not clearly demonstrate the fallopian tubes. HSG provides an accurate indication of tubal patency but employs ionizing radiation. It should therefore be performed in the first half of the menstrual cycle to avoid irradiating a patient with possible early pregnancy. Hysterosalpingo-ContrastSonography uses microbubbles to demonstrate the fallopian tubes and has the advantage of not using ionizing radiation or requiring anaesthesia but is less accurate than the other methods.

How well did you know this?
1
Not at all
2
3
4
5
Perfectly
55
Q

37 A 34 year old woman with 2 previous unremarkable vaginal deliveries presents with a history of pelvic pain. TV US shows diffuse heterogeneous myometrial echotexture with multiple tiny myometrial cystic lesions and poor definition of the junctional zone. MRI shows an indistinct diffuse predominantly low signal intensity lesion with small foci of increased signal intensity on T2W along with diffuse widening of the junctional zone on T2W. What is the most likely diagnosis?

(a) Adenomyosis

(b) Leiomyoma

(c) Endometrial carcinoma

(d) Muscular hypertrophy

(e) Myometrial contraction

A

(a) Adenomyosis

Adenomyosis is a term relating to benign invasion of the myometrium by endometrium. It can exist in either focal or diffuse forms. The diffuse form is characterised by junctional zone widening. Dilated cystic glands or haemorrhagic foci can be seen within the myometrium in 40% of diffuse cases.

How well did you know this?
1
Not at all
2
3
4
5
Perfectly
56
Q

54 With regards to normal ovarian anatomy on US, which of the following statements is false?

(a) Normal ovarian volume in a woman of reproductive age is up to 12 mis

(b) Normal ovarian volume in a postmenopausal woman is up to 4 mis

(c) The broad ligament contains the round ligaments within it

(d) Resistance to flow on Doppler imaging of the ovary is greatest in the luteal phase of the menstrual cycle

(e) The ovarian ligaments are not normally seen

A

(d) Resistance to flow on Doppler imaging of the ovary is greatest in the luteal phase of the menstrual cycle

Highest resistance to flow on Doppler is seen on days 1-8 of the menstrual cycle, gradually reducing with follicular development until day 21

How well did you know this?
1
Not at all
2
3
4
5
Perfectly
57
Q

67 With regards to fallopian tube recanalisation, which of the following statements is incorrect?

(a) It is used in cases of infertility associated with proximal tubal obstruction

(b) It cannot be employed after reversal of sterilization surgery

(c) Technical success is achieved in 80-90%

(d) It is associated with increased rates of ectopic pregnancy

(e) Salpingitis isthmica nodosa is a contraindication

A

(b) It cannot be employed after reversal of sterilization surgery

Although primarily used in cases of proximal tubal obstruction, it can be used to treat mid tubal obstructions after reversal of sterilization surgery.

How well did you know this?
1
Not at all
2
3
4
5
Perfectly
58
Q

72 A 69 year old woman presents with PV bleeding. US shows a normal sized uterus with an AP endometrial bilayer thickness of 16 mm. The maximally thickened endometrium is predominantly echogenic with scattered hypoechoic areas within it. At MRI a lesion of lower signal intensity than surrounding endometrium but higher signal intensity than myometrium on T2W is seen. Which of the following is the likeliest diagnosis?

(a) Adenomatous hyperplasia

(b) Submucosal leiomyoma

(c) Endometritis

(d) Endometrial carcinoma

(e) Benign endometrial hyperplasia

A

(d) Endometrial carcinoma

Other features to suggest malignancy include a resistive index of <0.7, disruption of the junctional zone or other evidence of myometrial invasion

How well did you know this?
1
Not at all
2
3
4
5
Perfectly
59
Q
  1. A 40 year old mother of three presents with menorrhagia and dysmenorrhoea. Transvaginal ultrasound shows an enlarged uterus with focal heterogeneous myometrial echotexture. The endometrium appears widened. T2-weighted MR imaging demonstrates focal widening of the junctional zone. There is a hypointense elongated myometrial mass with ill-defined margins. The mass contains foci of high signal on both T1- and T2-weighted imaging. The mass demonstrates contrast enhancement but to a lesser degree than the surrounding myometrium. What is the most likely diagnosis?

a. Leiomyoma

b. Endometrial carcinoma

c. Adenomyosis

d. Fibroma

e. Haematoma

A
  1. c. Adenomyosis

Adenomyosis is a focal or diffuse benign invasion of myometrium by endometrium, which incites reactive myometrial hyperplasia. It is associated with endometriosis (20–40%). It typically presents in multiparous women in the late reproductive years. Symptoms include pelvic pain, menorrhagia and dysmenorrhea, although adenomyosis it may be an incidental finding.
Adenomyosis may be diffuse or focal. Ultrasound appearances are variable but usually there is slight enlargement of the uterus with loss of homogeneity of the myometrium. There may be pseudo-widening of the endometrium due to increased myometrial echogenicity. MRI is more specific and demonstrates thickening of the junctional zone. When diffuse, a widened low-intensity junctional zone >12 mm confirms the diagnosis whereas <8 mm excludes the disease. For indeterminate sizes, further findings may aid the diagnosis, such as high-signalintensity linear striations extending out from the endometrium into the myometrium on T2 and high signal foci on T1 – representing ectopic endometrial tissue/haemorrhagic foci. When focal (adenomyoma), there is typically an oval/elongated mass with ill-defined margins residing within the myometrium which is in continuity with the junctional zone. Distinction from leiomyomas may be difficult but these tend to be round, sharply marginated masses occurring anywhere in the myometrium and they may contain calcifications.

How well did you know this?
1
Not at all
2
3
4
5
Perfectly
60
Q
  1. A 42 year old woman presents with post-coital bleeding. Transvaginal ultrasound shows the cervix to be enlarged, irregular and hypoechoic. MRI demonstrates a large cervical cancer with involvement of multiple pelvic lymph nodes. The left kidney is hydronephrotic. What is the most appropriate staging based on these findings?

a. T1

b. T2b

c. T3a

d. T3b

e. T4

A
  1. d. T3b

Cervical neoplasms are staged according to the TNM/FIGO classification. Stage I tumours are confined to the uterus. In stage IIA, there is involvement of the upper two-thirds of the vagina. Stage IIB shows parametrial invasion without pelvic sidewall involvement. Stage IIIA demonstrates invasion into the lower third of the vagina, and IIIB includes pelvic sidewall invasion with or without hydronephrosis. Tumour invasion into the bladder and rectal mucosa or distant metastasis accounts for stage IV disease. Pelvic nodal metastases do not alter the FIGO stage but para-aortic or inguinal node metastases are classified as stage IVB.

How well did you know this?
1
Not at all
2
3
4
5
Perfectly
61
Q
  1. A 50 year old woman presents with pelvic pain and abdominal fullness. Ultrasound reveals ascites and a large hypoechoic ovarian mass with posterior acoustic enhancement. CT demonstrates a well-defined solid pelvic mass which shows poor contrast enhancement. There is also a right-sided pleural effusion. Follow-up imaging postsurgical resection shows no residual tumour and resolution of ascites. What is the most likely diagnosis?

a. Serous cystadenocarcinoma

b. Mucinous cystadenocarcinoma

c. Ovarian fibroma

d. Brenner tumour

e. Massive ovarian oedema

A
  1. c. Ovarian fibroma

The condition described is Meigs syndrome. This occurs in about 1% of ovarian fibromas but is characterised by a large fibroma, ascites and a pleural effusion (typically right-sided). Ascites and effusion resolve after tumour resection. Fibromas are benign stromal tumours composed of fibrous tissue. On ultrasound they are typically solid hypoechoic lesions with posterior acoustic enhancement.

How well did you know this?
1
Not at all
2
3
4
5
Perfectly
62
Q
  1. A 28 year old woman presents with a dull ache in her pelvis. Ultrasound shows a 7 cm well-defined ovarian cyst. A distinct echogenic nodule which causes dense acoustic shadowing is seen projecting into the cyst’s lumen. What is the most likely diagnosis?

a. Mature cystic teratoma

b. Tubo-ovarian abscess

c. Endometrioma

d. Ovarian carcinoma

e. Corpus luteum cyst

A
  1. a. Mature cystic teratoma

Mature cystic teratomas (dermoid cysts) account for approximately 15% of all ovarian tumours. They are benign germ cell tumours containing tissues from all three germ cell layers. They most commonly present in younger women of reproductive age (20–40 years) and may be bilateral in up to 25%. They are generally cystic masses that may contain a pathognomonic distinct hyperechoic mural nodule (dermoid plug/Rokitansky nodule) which projects into the cystic lumen and causes posterior acoustic shadowing. This nodule represents in-growth of solid tissue such as hair or teeth from the tumour wall.

How well did you know this?
1
Not at all
2
3
4
5
Perfectly
63
Q
  1. A 23 year old woman undergoes investigation for dyspareunia. Pelvic ultrasound was unremarkable. MRI demonstrates a 1 cm thin-walled ovoid cystic lesion at the anterolateral aspect of the upper vagina. It is homogeneously hypointense on T1 and shows marked hyperintensity on T2. What is the most likely diagnosis?

a. Bartholin cyst

b. Nabothian cyst

c. Cervical fibroid

d. Gartner duct cyst

e. Cervical polyp

A
  1. d. Gartner duct cyst

Gartner’s duct cysts are remnants of mesonephric ducts and have a reported incidence of 1–2%. They are ovoid, thin-walled cysts located at the anterolateral aspect of the upper vagina and generally measure less than 2 cm. They may contain proteinaceous material, making them slightly hyperintense on T1. They may be associated with Herlyn–Werner– Wunderlich syndrome (ipsilateral renal agenesis and ipsilateral blind vagina) and ectopic ureter inserting into the cyst. Bartholin cysts are located at the lateral introitus adjacent to the labia minora. Nabothian cysts are epithelial inclusion cysts which develop in the endocervical canal and are most commonly found in the perimenopausal period. Cervical fibroids and cervical polyps show mainly as solid lesions.

64
Q
  1. A 48 year old woman undergoes investigation for postmenopausal bleeding. Ultrasound shows a hyperechoic endometrial mass which contains several small cystic spaces. Power Doppler reveals a vessel at its base. On T2-weighted MR imaging the mass contains a central fibrous core with low signal intensity and small, welldelineated cysts showing marked high signal intensity. The central core enhances post-contrast administration. The junctional zone is intact. What is the most likely diagnosis?

a. Endometrial hyperplasia

b. Submucosal leiomyoma

c. Submucosal fibroid

d. Adenomyoma

e. Endometrial polyp

A
  1. e. Endometrial polyp

Endometrial polyps are common benign tumours of the endometrial cavity. They are most common after the age of 40 years and are rare before menarche. Typical ultrasound appearance is of a hyperechoic endometrial mass which may or may not contain cystic spaces. A feeding vessel is often demonstrated from its base on power Doppler. (Submucosal fibroids are generally of reduced echogenicity). On MRI, a mass which contains a central fibrous core that enhances post-contrast and also contains well-demarcated T2-hyperintense cysts suggests endometrial polyp. An intact junctional zone and smooth tumour-myometrium interface also favour a polyp.

65
Q
  1. A 19 year old female presents with vague lower abdominal pain. Ultrasound shows a right 5 cm thin-walled unilocular ovarian cyst. Follow-up ultrasound six weeks later shows cyst regression. What is the likely diagnosis?

a. Corpus luteum cyst

b. Endometrioma

c. Serous cystadenoma

d. Surface epithelial inclusion cyst

e. Follicular cyst

A
  1. e. Follicular cyst

These are common ovarian masses that result from a failure of the mature Graafian follicle to rupture and release ova. Typically, they are smooth, thin-walled, unilocular anechoic cysts that show spontaneous regression within four to six weeks. They may undergo haemorrhagic change producing internal echogenic material. They are generally larger than 2.5 cm and may occasionally grow up to 10 cm.

66
Q
  1. A female patient undergoes investigation for dysmenorrhea. She is obese, hirsute and has elevated luteinising hormone levels. Which of the following ultrasonographic findings is consistent with a diagnosis of polycystic ovarian syndrome?

a. Ovarian volume >10 ml when no follicles measuring over 5 mm in diameter are present

b. Ten or more follicles (3–12 mm diameter) present in an ovary

c. Ovarian volume >15 ml when no follicles measuring over 10 mm in diameter are present

d. Twelve or more follicles (3 mm diameter) present in an ovary

e. Ovarian volume >10 ml when no follicles measuring over 10 mm in diameter are present

A
  1. e. Ovarian volume >10 ml when no follicles measuring over 10 mm in diameter are present

Diagnosis of polycystic ovary syndrome should not be made on imaging findings alone; clinical and biochemical studies must be obtained. The diagnosis can be supported when one or more of the following ultrasonographic features are demonstrated:Twelve or more follicles (3–12 mm diameter) are present in an ovary (either peripheral or diffusely arranged).Ovarian volume >10 ml when no follicles measuring over 10 mm in diameter are present. If a follicle >10 mm is present then the volume should be recalculated on a repeat scan when the ovary is quiescent to prevent overestimation of the ovarian volume.

67
Q
  1. A 14 year old girl presents with acute onset of right lower abdominal pain. She reports that she has had similar symptoms previously. Ultrasound shows an ovoid-shaped enlarged right-sided ovary containing multiple enlarged follicles. The ovarian stroma is echogenic compared to adjacent myometrium. There is peripheral blood flow on power Doppler and free fluid within the pelvis. What is the most likely diagnosis?

a. Ovarian hyperstimulation

b. Ovarian torsion

c. Polycystic ovary syndrome

d. Theca lutein cysts

e. Serous cystadenoma

A
  1. b. Ovarian torsion

Ovarian torsion usually presents in the first three decades of life and is predisposed inpatients with co-existing ovarian pathology such as follicular cyst. There may be history of similar episodes indicating intermittent torsion and spontaneous detorsion. Torsion causes venous outflow obstruction and engorgement of the ovary. Eventually arterial supply is compromised and necrosis ensues. Diagnosis is suggested by unilateral enlargement of a round or oval-shaped ovary containing multiple enlarged peripheral cysts (caused by transudation of fluid into follicles). Free fluid is present in the majority of cases. Peripheral blood flow may be present but may be absent with infarction. Ovarian hyperstimulation can present with abdominal pain and may show an enlarged multicystic ovary associated with ascites. However, the condition usually arises from ovarian hormone stimulation in the setting of infertility. Polycystic ovary syndrome typically presents with menstrual disturbance, obesity and hyperandrogenism

68
Q
  1. A 38 year old female undergoes investigation for weight loss and abdominal fullness. CT shows large bilateral adnexal masses, ascites and several small omental soft-tissue nodules. MRI demonstrates bilateral sharply marginated ovarian tumours with preservation of the ovarian contours. The tumours consist mainly of hypointense solid material interspersed with foci of high-signal cysts. On post-contrast T1-weighted imaging the solid components are hyperintense. What is the most likely diagnosis?

a. Cystadenocarcinoma

b. Dysgerminoma

c. Krukenberg’s tumour

d. Burkitt’s lymphoma

e. Granulosa cell tumour

A
  1. c. Krukenberg’s tumour

Krukenburg’s tumours are metastatic tumours of the ovary. The colon and stomach are the most common primary tumour sites, but other sites, such as the breast, lung and pancreas, have also been reported. They display characteristic imaging features, including bilateral, sharply marginated oval tumours which preserve the contour of the ovary. Identification of hypointense solid components on T2-weighted imaging corresponding to areas of dense collagenous stroma is also considered characteristic.

69
Q
  1. A five year old girl presents with atypical genital bleeding, breast development and pubic hair growth. T2-weighted MR imaging demonstrates a large solid mass with high signal intensity and an enlarged uterus with thick endometrium. Ascites is also present. Post-gadolinium T1-weighted imaging shows homogeneous tumour enhancement. What is the most likely diagnosis?

a. Immature teratoma

b. Sertoli–Leydig cell tumour

c. Thecoma

d. Sclerosing stromal tumour

e. Granulosa cell tumour

A
  1. e. Granulosa cell tumour

Granulosa cell tumours are the most common ovarian tumours with oestrogenic manifestations that are classified as sex-cord-stromal tumours. They are subdivided into adult and juvenile types. The juvenile form affects prepubertal children and causes pseudoprecocity. In about a third of cases Sertoli–Leydig cell tumours cause virilisation. Thecomas are oestrogen-producing tumours but more than 80% occur in postmenopausal women. Immature teratomas are extremely rare but do occur in children. Elevated alpha-fetoprotein is found in up to 65% of cases. Sclerosing stromal tumours usually affect women younger than 30 years of age and a few cases have shown androgenic or oestrogenic manifestations. They are also known as hypervascular tumours which show early peripheral enhancement with centripetal progression.

70
Q
  1. A 23 year old female presents with acute lower abdominal pain. She has been sexually active since the age of 15 years. Ultrasound shows a well-defined, oval-shaped, relatively thin-walled, anechoic fluid-filled structure lying adjacent to the left lateral wall of the uterus. The mass appears septated although the septae do not fully cross the lumen. What is the most likely diagnosis?

a. Hydrosalpinx

b. Tubo-ovarian abscess

c. Haemorrhagic ovarian cyst

d. Endometrioma

e. Thrombosed ovarian vein

A
  1. a. Hydrosalpinx

Hydrosalpinx describes a fallopian tube filled with fluid. The fluid is most often anechoic. When the fluid becomes infected, the term pyosalpinx is used and the fluid contents tend to be echogenic. Hydro/pyosalpinx appear as tortuous, well-defined, fluid-filled, oval-shaped structures which extend from the cornua to the ovaries. They are often mistaken for multicystic adnexal masses or septated ovarian cysts due to apparent internal septations. However, the septa, actually the folded wall of the fallopian tube, do not cross the lumen completely. Hydro/pyosalpinx occur most commonly as a result of acute salpingitis and pelvic inflammatory disease (history of early sexual activity/multiple sexual partners). They have also been reported following pelvic surgery. Tubo-ovarian abscesses tend to be multilocular, irregular, thick-walled, complex masses containing debris and internal septations. Internal fluid-fluid levels or gas may also be seen.

71
Q
  1. A 13 year old girl presents with lower abdominal pain. She says she has had it intermittently for over a year. She has not yet had a period. On ultrasound examination the uterus is displaced cranially by a large cystic mass in the region of the vagina. It contains a large quantity of echogenic fluid and a fluid-debris level is visible. The bladder is not visualised. What is the most likely diagnosis?

a. Duplication cyst

b. Rectovesical fistula

c. Haematocolpos

d. Hydrometra

e. Cloacal malformation

A
  1. c. Haematocolpos

Haematocolpos is the accumulation of blood within the vagina and is typically caused by an imperforated hymen. This causes acute-on-chronic lower abdominal/pelvic pain as menstrual blood is prevented from normal discharge (apparent lack of menstruation). Ultrasound reveals an echogenic cystic mass with or without fluid-debris levels in the region of the vagina. The distended vagina often causes displacement of the uterus and compression of the bladder so that the latter may not be visualised. Cloacal malformation is a single perineal orifice for the bladder, vagina and rectum caused by early embryonic arrest. It manifests in the newborn period. Hydrometra is fluid within the uterus and may be due to cervical or vaginal dysgenesis. Rectal duplication cysts may reveal an echogenic cystic mass in childhood but they often present with constipation and faecal soiling.

72
Q
  1. A female undergoes transvaginal ultrasound for postmenopausal bleeding. In which of the following situations can you virtually exclude the presence of endometrial cancer?

a. An endometrial thickness of 5 mm in a patient who has never undergone hormone replacement therapy (HRT)

b. An endometrial thickness of 6 mm in a patient using sequential combined HRT

c. An endometrial thickness of 5 mm in a patient using continuous combined HRT

d. An endometrial thickness of 4 mm in a patient using sequential combined HRT

e. An endometrial thickness of 4 mm in a patient who has not used any form of HRT for one year or more

A
  1. d. An endometrial thickness of 4 mm in a patient using sequential combined HRT

An endometrial thickness of 3 mm can be used to exclude endometrial cancer in women who: Have never used HRT, or Have not used any form of HRT for one year, or Are using continuous combined HRT. In the above conditions the post-test risk of a patient having endometrial cancer is 0.6–0.8% when the endometrial thickness is 3 mm but 20–22% when the endometrial thickness is >3 mm. An endometrial thickness of 5 mm can be used to exclude endometrial cancer in women using sequential combined HRT (or having used it within the past year). In this scenario the post-test risk of a patient having endometrial cancer is 0.1–0.2% when the endometrial thickness is 5 mm but 2–5% when the endometrial thickness is >5 mm.

73
Q
  1. A 25 year old woman presents with an eight-month history of intermittent lower abdominal pain. Ultrasound demonstrates a 4 cm complex mass related to her left ovary. On MRI the mass has predominantly high signal on T1- and T2-weighted imaging and T2- weighted fat-suppressed sequences. The most likely diagnosis is:

a. Endometriosis

b. Follicular cyst

c. Cystadenocarcinoma

d. Dermoid cyst

e. Tubo-ovarian abscess

A
  1. a. Endometriosis

The ovaries are the most common site for endometriosis, accounting for greater than 80%. Other sites include the uterosacral ligaments, pouch of Douglas, uterine serosal surface, fallopian tubes and rectosigmoid. MRI is more specific than either CT or ultrasound in its detection. Typically, MR demonstrates a homogeneously hyperintense cyst due to the presence of methaemoglobin, which shortens T1. It is also hyperintense on fat-suppressed T2-weighted imaging, which virtually excludes a dermoid cyst. On T2-weighted imaging there may be faint or complete loss of signal. This phenomenon is referred to as ‘shading’ and results from the high protein and iron concentration from recurrent haemorrhage into the endometrioma.

74
Q

QUESTION 22
A 63-year-old nulliparous, obese female patient presents with postmenopausal bleeding. On a transvaginal ultrasound examination the endometrium is 8 mm thick. The radiology SpR suspects endometrial carcinoma and suggests an MRI for further evaluation. What MRI findings would support a diagnosis of endometrial carcinoma?

A On post-gadolinium Tlw images, the thickened endometrium enhances less than the adjacent myometrium.

B On Tlw sequences, the thickened abnormal endometrium appears hypointense to the adjacent myometrium.

C Post-gadolinium fat-suppressed Tlw sequences show enhancement of the thickened endometrium.

D Tlw sequences show the junctional zone as a low signal band sandwiched between the high signal endometrium and the intermediate signal peripheral myometrium.

E T2w sequences show the junctional zone as a high signal band.

A

A On post-gadolinium Tlw images, the thickened endometrium enhances less than the adjacent myometrium.

75
Q

QUESTION 26
A 36-year-old woman undergoes a pelvic MRI following the discovery of a left ovarian mass on ultrasound. The reporting radiologist identifies a 3-cm mass in the left ovaiy and believes this to be an ovarian fibroma. Which one of the following statements best describes the likely MRI findings?

A A left ovarian mass of high signal on Tlw and low signal on T2w images

B A left ovarian mass of high signal on Tlw, showing loss of signal on STIR images

C A left ovarian mass of high signal on T2w but low signal on Tlw images

D A left ovarian mass yielding low signal on both Tlw and T2w images

E Compared with in-phase images, the 3-cm ovarian mass shows signal loss in out-of-phase Tlw sequences

A

D A left ovarian mass yielding low signal on both Tlw and T2w images

Ovarian fibromas have characteristic low signal on both Tlw and T2w sequences, due to the presence of densely packed connective tissue.

76
Q

QUESTION 40
A 36-year-old woman presents with primary infertility. A transabdominal ultrasound shows a normal anteverted uterus and bilateral adnexal masses. A subsequent MRI shows bilateral high signal ovarian masses on both Tlw and T2w sequences. On fat-suppressed Tlw images, the lesions remain high signal. What is the most likely diagnosis?

A Bilateral dermoid cysts

B Bilateral endometriomas

C Bilateral ovarian fibromas

D Bilateral theca lutein cysts

E Polycystic ovaries

A

B Bilateral endometriomas

Theca lutein cysts contain straw-coloured fluid which is low signal on Tlw and high signal on T2w images

77
Q

QUESTION 44
A 45-year-old woman presents with a 1-year history of menorrhagia. On physical examination, her GP palpates a mass arising from the pelvis and suspects uterine fibroids. The GP refers her for imaging to confirm the diagnosis of uterine fibroids. Which one of the following radiological findings would support this diagnosis?

A On T2w images, a well-circumscribed myometrial mass that is of lower signal than the surrounding myometrium

B On ultrasound, multiple theca lutein cysts and an enlarged uterus that contains multiple cystic areas

C On Tlw images, a well-circumscribed high signal intensity mass arising from the myometrium

D On postcontrast Tlw images, a well-circumscribed, uniformly enhancing myometrial mass that is of higher signal than the surrounding myometrium

E Widened junctional zone on Tlw images of the uterus.

A

A On T2w images, a well-circumscribed myometrial mass that is of lower signal than the surrounding myometrium

78
Q

@#e QUESTION 45
A transvaginal ultrasound is performed on a 36-year-old woman with dysfunctional uterine bleeding. This demonstrates an enlarged globular uterus with a heterogeneous appearance of the myometrium. The myometrium contains diffuse echogenic nodules, subendometrial echogenic linear striations and 2- to 6-mm subendometrial cysts. Colour Doppler demonstrates a speckled pattern of increased vascularity within the heterogeneous area of myometrium. What is the most likely diagnosis?

A Adenomyosis

B Endometrial polyposis

C Gestational trophoblastic.disease (GTD)

D Stage 1A endometrial cancer

E Uterine fibroid

A

A Adenomyosis

79
Q

@#e A 60-year-old nulliparous woman presents with postmenopausal bleeding. On transvaginal ultrasound, her endometrium is 8 mm thick and the endomyometrial junction appeared indistinct. The radiologist suspects invasive endometrial cancer and refers her for an MRI examination. What are the likely findings on MRI?

A On unenhanced Tlw images the endometrial cancer appears of high signal intensity compared to the surrounding myometrium.

B On contrast-enhanced Tlw images, endometrial cancer shows avid enhancement compared with surrounding myometrium.

C On T2w images the normally high signal junctional zone is disrupted.

D Tlw fat-saturated sequences are best used to assess the junctional zone.

E The endometrial cancer demonstrates delayed/little enhancement compared to the normal surrounding myometrium on postcontrast Tlw images

A

E The endometrial cancer demonstrates delayed/little enhancement compared to the normal surrounding myometrium on postcontrast Tlw images

80
Q

QUESTION 69
A 22-year-old woman presents to her GP with irregular menstrual periods. She is overweight with a body mass index of 32 and has excess body hair. Her LH/FSH ratio is elevated and her GP refers her for a pelvic ultrasound. Which one of the following findings are most likely to be present on ultrasound?

A Enlarged, oedematous ovaries with multiple packed follicles and pelvicfree fluid

B Enlarged ovaries with multiple peripheral cysts

C Normal appearances of the ovaries D Ovarian mass with mixed cystic and solid components

E Ovaries replaced by multiple large cysts

A

B Enlarged ovaries with multiple peripheral cysts

In polycystic ovarian syndrome, the ovaries are enlarged with an echogenic central stroma and more than ten peripherally placed cysts (each less than 9 mm in diameter). Option A describes ovarian hyperstimulation syndrome

81
Q

@#e QUESTION 73
A 64-year-old woman presents with bloating and vague pelvic pain and is referred for a pelvic ultrasound. On transabdominal ultrasound, she is found to have a large right adnexal mass. Which one of the following sonographic findings would indicate that this mass is more likely to be malignant than benign?

A Doppler waveform with a high resistive index (> 0.8)

B Homogeneously hypoechoic mass with posterior acoustic enhancement

C Multiple septations that are approximately 1 mm thick

D Papillary projections

E Size > 4 cm

A

D Papillary projections

82
Q

@#e QUESTION 74
A 31-year-old woman has a hysterosalpingogram (HSG) as part of a series of investigations for primary infertility. The HSG shows a single vagina, single cervix but two separate uterine cavities leading to separate uterine horns. What is the most likely diagnosis?

A Arcuate uterus

B Bicornuate uterus

C Didelphus uterus

D Septate uterus

E Bicornis bicollis

A

B Bicornuate uterus

83
Q
  1. A 45-year-old women presents with menorrhagia and dysmenorrhea. She has had three successful pregnancies and one therapeutic abortion in the past. She undergoes an MRI of the pelvis 14 days after the start of her last menstrual period. It reveals a junctional zone which measures 13 mm throughout, with hyperintense T2WI foci within it. With what conditions are these findings most consistent?

A. Endometrial hyperplasia.

B. Endometrial carcinoma stage 1A.

C. Pseudothickening.

D. Adenomyosis.

E. Myometrial contraction.

A
  1. D. Adenomyosis.

This appears as focal or diffuse thickening of the junctional zone of 12 mm or greater. Thickening of 8–12 mm is indeterminate, while thickening less than 8 mm usually allows exclusion of the disease. The bright foci on T2WI correspond to islands of ectopic endometrial tissue and cystic dilatation of glands, and have been reported to be present in up to 50% of cases of adenomyosis. Occasionally haemorrhage within these areas of ectopic endometrial tissue can result in areas of high signal within the junctional zone on T1WI. Pseudothickening of the junctional zone can mimic adenomyosis. It occurs in the menstrual phase (especially the first 2 days) and thus scanning should not be performed at this time. Myometrial contractions can also mimic adenomyosis, but they result in focal, rather than diffuse, thickening of the junctional zone. The contractions are transient and thus will not persist on different sequences. They can therefore be more easily detected with multiphase–multisection imaging, rather than static fast SE. Neither pseudothickening nor myometrial contractions alone would result in high T2WI foci. The appearances are not consistent with endometrial carcinoma or hyperplasia.

84
Q
  1. A 62-year-old woman presents with recurrent urinary tract infections (UTIs) and a pelvic/renal tract ultrasound is performed. This demonstrates normal kidneys and bladder, but there is a 5-cm solid, hypoechoic mass seen arising from the right ovary. There is acoustic shadowing caused by the mass, but a subsequent CT scan does not show any calcification within the mass or any metastatic disease. What is the most likely cause of the ovarian mass?

A. Sertoli–Leydig cell tumour of the ovary.

B. Cystadenocarcinoma of the ovary.

C. Fibrothecoma of the ovary.

D. Granulosa cell tumour of the ovary.

E. Embryonal cell carcinoma of the ovary.

A
  1. C. Fibrothecoma of the ovary.

oiOvarian fibroma is the most common sex-cord stromal neoplasm and is almost always benign. Occasionally there will be histologic features of both fibroma and thecoma, giving rise to the term ‘fibrothecoma’. These are typically solid lesions on ultrasound and have marked acoustic shadowing in 18–52% of cases. This shadowing is not secondary to calcification, but is related to marked attenuation of sound by the dense hypoechoic mass itself. This can be a useful sign for identifying fibrothecoma on ultrasound. Classification of ovarian neoplasms is based on histologic features and typically includes the general categories of epithelial, germ cell, sex cord-stromal, and metastatic neoplasms. Cystadenomas/adenocarcinomas belong to the epithelial group of malignancies and make up over 90% of primary ovarian malignancies. Sertoli–Leydig cell tumours, granulosa cell tumours, and fibrothecomas belong to the sex cordstromal group of neoplasms. Sertoli–Leydig cell tumours may be subtle on ultrasound, as many are small and solid. They can secrete androgens and cause virilization. Granulosa cell tumours often have a sponge-like consistency and may produce oestrogen and cause endometrial disease. Dysgerminomas and embryonal cell carcinomas are very rare germ cell neoplasms. The most common germ cell neoplasm is of course a mature cystic teratoma (dermoid cyst). A hyperechoic area (not usually as intensely echogenic as calcification) with acoustic shadowing is highly predictive of this lesion. Hyperechoic lines and dots, sometimes known as the dermoid mesh, are also highly predictive of a mature teratoma. Less common associated signs include a fluid–fluid level and floating globules. Calcification can be seen, but is not specific to dermoids, as other ovarian neoplasms can calcify

85
Q
  1. A 43-year-old female presents with pelvic pain. On examination she is tender in the left iliac fossa and midline. Inflammatory markers are normal. A trans-abdominal ultrasound reveals a normal right ovary and uterus, but a 5.5-cm simple appearing cyst arising from the left ovary. As the reporting radiologist you:

A. refer to gynaecology for clinical assessment and serum Ca-125 measurement.

B. recommend trans-vaginal ultrasound.

C. recommend a repeat trans-abdominal ultrasound in 6 weeks’ time.

D. recommend MRI of pelvis.

E. issue a report stating that a benign simple cyst is seen to arise from the left ovary.

A
  1. C. recommend a repeat trans-abdominal ultrasound in 6 weeks’ time.

At ultrasound, a functional ovarian cyst is typically anechoic with a thin, smooth wall and posterior acoustic enhancement. Regardless of size, these cysts are unlikely to be malignant. A unilocular cyst greater than 3 cm in a premenopausal or greater than 5 cm in a postmenopausal female patient should be rescanned in approximately 6 weeks (which will usually be at a different stage of the menstrual cycle in the case of premenopausal patients). Premenopausally this is the case, even if there are some internal echoes (but not other findings raising the possibility of malignancy), as these quite often represent haemorrhage within a cyst. In the premenopausal patient, the follow-up scan is to ensure there is reduction in size or resolution of what is usually a benign finding and commonly merely a dominant follicle; in the postmenopausal patient, it is to ensure there is no interim enlargement. If the cyst is smaller than the above dimensions (i.e. <3 cm pre and <5 cm post menopausal) then, in the absence of other sinister findings or history, it can be assumed that the cysts are benign. Sonographic findings which increase the likelihood of malignancy include wall thickening, solid elements, septations, blood flow within solid elements, abnormal blood flow, and findings of metastatic disease. If a cyst displays any of the latter features or increases in size on interval scanning, then options A, B, and D may be valid steps in management

86
Q
  1. A 25-year-old female undergoes a hysterosalpingogram (HSG) that reveals a unicornuate uterus. This is confirmed on MRI, which also demonstrates a non-functioning rudimentary contra-lateral horn. What further investigation is indicated?

A. Laparoscopy.

B. Renal imaging.

C. Pelvic ultrasound.

D. Laparotomy and resection of rudimentary horn.

E. Speculum examination

A
  1. B. Renal imaging.

Mullerian duct anomalies are associated with renal tract anomalies in up to 30% of cases due to the close embryologic relationship between the Mullerian and mesonephric ducts. The most common renal tract abnormality is renal agenesis. Other associated anomalies include duplicated collecting system, renal duplication, horseshoe kidney, crossed renal ectopy. and cystic renal dysplasia. Renal imaging is therefore essential in all patients with Mullerian duct anomalies. Surgical intervention is not required in patients with a unicornuate uterus and a non-functioning rudimentary horn.

87
Q

(Ped) 14. A 15-year-old female undergoes a pelvic MRI on which incidental note is made of a 1.5-cm lesion antero-lateral to the vagina and above the level of the inferior margin of the pubic symphysis. The lesion is hypointense on T1WI and hyperintense on T2WI. There is no displacement of or communication with the urethra. What is the likely diagnosis?

A. Urethral diverticulum.

B. Gartner duct cyst.

C. Bartholin gland cyst.

D. Skene duct cyst.

E. Urethral caruncle

A
  1. B. Gartner duct cyst.

Anatomical location is useful in the differential diagnosis of periurethral cysts. A Gartner duct cyst is a retention cyst arising from Wolffian (mesonephric duct remnant). It is typically located in the antero-lateral aspect of the vagina above the level of the pubic symphysis. It is usually solitary and less than 2 cm in size. At MRI, the lesion may appear hypo- or hyperintense on T1WI depending on whether the contents are simple fluid or haemorrhagic/proteinaceous. They do not displace or deform the urethra.
A Bartholin cyst is located in the posterolateral aspect of the lower vagina, at or below the level of the pubic symphysis. Skene cysts are located lateral to the external urethral meatus, below the level of pubic symphysis. It may be difficult to differentiate Bartholin and Skene cysts. Urethral diverticula communicate with the urethra whereas periurethral cysts do not. A urethral caruncle manifests as T2 hyperintense soft tissue surrounding the external urethral meatus.

88
Q
  1. A 36-year-old woman undergoes MRI of the pelvis for assessment of pelvic pain. She had a previous hysterectomy due to post-partum haemorrhage and thus transvaginal ultrasound (TVUS) is not an option. Abdominal ultrasound is technically difficult due to body habitus and the ovaries cannot be visualized. MRI reveals a 7-cm left adnexal lesion of predominantly intermediate and high signal on T2WI, but with low-signal components within. On T1WI, there is layering of low and high signal, with suppression of the high signal on the T1WI with fat saturation. What do you advise in your report?

A. Referral for chemoradiotherapy. B. Referral for laparoscopy for staging.

C. Follow up in 3 months’ time.

D. Referral for surgery.

E. No follow-up required.

A
  1. D. Referral for surgery.

The findings are consistent with a mature cystic teratoma or dermoid cyst. They are derived from all three germ cell layers and thus contain fat and may contain desquamated epithelium, skin, hair, and teeth. The classical finding is of high T1WI signal, which suppresses on fat saturation imaging. They can often have a fat–fluid level. The low signal on T2WI may represent tooth or calcification. Soft-tissue protuberances represent Rokintansky nodules or ‘dermoid plugs’ of sebaceous material. It is the most common ovarian tumour, accounting for 20% of all ovarian neoplasms. Dermoids can be bilateral in 8–25% of cases. The chance of malignant transformation is low but surgical resection is indicated, not just for relief of symptoms, but because of the risk of torsion (in 4–16% of cases) or rupture (the latter is rare but can lead to chemical peritonitis). Adjuvant treatment need not be considered unless the case is complicated by malignant transformation.

89
Q

(Ped) 33. A 5-year-old girl with a history of precocious puberty and increased serum inhibin levels is referred for ultrasound of the pelvis. On ultrasound, there is a complex solid/cystic mass in the adnexa. MRI of the pelvis confirms a solid/cystic ovarian mass with a ‘sponge-like appearance’ on T2WI. What is the likely diagnosis?

A. Sertoli–Leydi cell tumour.

B. Juvenile granulosa cell tumour.

C. Mature cystic teratoma.

D. Fibroma.

E. Mucinous cystadenocarcinoma.

A
  1. B. Juvenile granulosa cell tumour.

This is a sex cord stromal tumour arising from the granulose-thecal cells. They typically affect prepubescent girls and are unilateral. They are usually hormonally active (secrete oestrogen), resulting in precocious puberty. Granulosa cell tumours have a varied appearance on imaging. They may be completely solid with or without haemorrhagic or fibrotic changes, multilocular solid/cystic, or a completely cystic neoplasm. On T2WI they have a characteristic sponge-like appearance with innumerable cystic areas in a solid lesion of intermediate signal. Inhibin is the tumour marker used to monitor these lesions.
Sertoli–Leydig cell tumour is also hormonally active, but it results in virilization and hirsutism. Malignant epithelial neoplasms of the ovaries are extremely rare in prepubertal girls. Mature cystic teratoma is the most common type of ovarian neoplasm in the paediatric age group, containing derivatives of at least two of the three germ cell layers: ectoderm, mesoderm, or endoderm. Fibroma is rare in children.

90
Q
  1. A 23-year-old woman is referred for an MRI of pelvis because of dyspareunia and pelvic pain. The ovaries are normal, but a 6-mm rounded area of high signal on T1WI and T2WI sequences is demonstrated in the left posterolateral aspect of the distal vagina. What is the most likely diagnosis?

A. Squamous cell carcinoma of the vagina.

B. Urethral diverticulum.

C. Nabothian cyst.

D. Bartholin’s gland cyst.

E. Vaginal septum.

A
  1. D. Bartholin’s gland cyst.

Bartholin’s glands are located behind the labia minora and their ducts open onto the posterolateral vestibules on each side. The cysts are due to blockage of the ducts and retention of secretions. They are common in reproductive age and are usually cystic in appearance (high T2WI signal). Their T1WI signal is variable and can be high due to mucoid content. They are usually discovered as incidental findings on imaging for another cause, although they may be palpable or clinically visible. The radiologist should recognize them as a benign pathology. Primary carcinoma of the vagina is rare, as 80–90% of tumours affecting the vagina are due to extension of primary bladder, cervical, vulval, or rectal tumours. Ninety per cent of primary carcinomas are squamous cell carcinoma. The majority occur in elderly women and the two most common presenting complaints are vaginal discharge and bleeding. Squamous cell carcinomas of the vagina are usually of low T1WI and intermediate T2WI signal. As for vulval and cervical squamous cell carcinoma, human papilloma virus (HPV) infection is a risk factor. Urethral divertcula are best seen on T2WI sequences as hyperintense lesions adjacent to or surrounding the urethra: a connection to the urethra is not always seen. They are usually of low signal on T1WI sequences. They predispose to stones (low signal on T2WI sequences) and carcinoma (enhancing on T1W fat saturation post gadolinium). Nabothian cysts are found at the external os of the cervix. They are benign retention cysts that develop secondary to obstruction of endocervical glands. They have high T2WI signal and may have high T1WI signal due to mucinous content. They are usually multiple and measure less than 2 cm. They become clinically relevant when they are mistakenly diagnosed as adenoma malignum, a subtype of mucinous adenocarcinoma of the cervix. A vaginal septum is a low-signal T2WI band. If transverse, it may present in adolescence with primary amenorrhea and abdominal pain, and an abdominal mass if complete. A longitudinal septum is usually asymptomatic and is associated with duplication of the uterus, cervix, or vagina.

91
Q
  1. A 45-year-old woman presents with anorexia and loss of weight. A CT of abdomen reveals bilateral non-calcified ovarian masses, but no ascites and no significant pelvic lymphadenopathy or peritoneal disease. Serum alpha fetoprotein, beta human chorionic gonadotrophin (b-HCG), and CA-125 are normal. The gynaecology team request an MRI of pelvis. This reveals that both ovaries are enlarged, the left more than the right. Bilateral cystic masses with low signal on T2WI/intermediate signal T1WI with solid portions are demonstrated. Gadolinium-enhanced T1WI with fat saturation reveals marked enhancement of these solid components and septa. What is the most likely diagnosis?

A. Krukenberg tumours.

B. Sertoli–Leydig tumours.

C. Dysgerminomas.

D. Fibromas.

E. Endodermal sinus tumours

A
  1. A. Krukenberg tumours.

The history points towards a GI source, namely gastric carcinoma. Approximately 5–15% of ovarian tumours are metastatic lesions. They are bilateral in 75% of cases. They are often asymptomatic and may present before the primary tumour. They should be considered, along with serous epithelial tumours of the ovary, when bilateral complex ovarian masses are demonstrated. Krukenberg tumours represent ovarian metastases that contain mucin-secreting ‘signet-ring’ cells from colonic or gastric neoplasms. Other primary neoplasms that less commonly metastasize to the ovary are breast, lung, and the contra-lateral ovary (not strictly Krukenberg tumours due to the cell types). Imaging findings in metastatic lesions are nonspecific, consisting of predominately solid components or a mixture of cystic and solid areas. In Krukenberg tumours, there are distinctive findings, including solid components secondary to stromal reaction that are low T2WI and high T1W signal. A Sertoli–Leydig tumour is a sex cord-stromal tumour that occurs in young women (less than 30 years old). It manifests as a well-defined, enhancing solid mass with intra-tumoral cysts. They constitute 0.5% of ovarian tumours. They are rarely bilateral. Dysgerminomas are rare ovarian tumours that occur predominantly in young women. Serum E-HCG is increased in 5%. Calcification may be present in a speckled pattern. Characteristic imaging findings include multilobulated solid masses with prominent fibrovascular septa. They are rarely if ever bilateral. Fibromas are benign solid tumours of the ovary, which are of low signal on T1W and very low signal on T2W sequences. They can be associated with ascites and pleural effusions (Meigs syndrome), and dense calcifications are often seen. Together with fibrothecomas they form a spectrum of benign tumours. Endodermal sinus tumour, also known as yolk sac tumour, is a rare malignant ovarian tumour that usually occurs in the second decade of life. These tumours manifest as a large, complex pelvic mass that extends into the abdomen and contains both solid and cystic components. They are rarely bilateral. They grow rapidly and have a poor prognosis. Affected patients have an elevated serum alpha feta protein.

92
Q
  1. A 60-year-old woman presents with pelvic pain. An MRI reveals multiple large areas of well-circumscribed, predominantly homogenous low T2WI and low/intermediate T1WI signal within the myometrium. There are also a number of peritoneal nodules demonstrated. What is the most likely diagnosis?

A. Endometriosis.

B. Adenomyosis.

C. Uterine leiomyomas.

D. Endometrial carcinoma.

E. Metastases to the uterus.

A
  1. C. Uterine leiomyomas.

Also known as uterine fibroids, these are very common. They are usually of low signal on T2WI and intermediate signal on T1WI. Benign metastasizing leiomyoma is an unusual variant, with tumours in the lungs, lymph nodes, or peritoneal nodules, not uncommonly from a benign uterine fibroid, which has been removed many years earlier. Malignant transformation to leiomyosarcoma is very rare and it is thought the latter may arise de novo even in the presence of leiomyomas. Endometrial sarcoma might produce peritoneal metastases, but would be expected to show a large heterogenous mass with indistinct or irregular borders arising from the endometrial canal. Endometriosis and adenomyosis are diseases of the reproductive years. Metastases to the uterus are rare and if present represent advanced malignancy. There is usually diffuse enlargement of the myometrium, which can be of low or high signal.

93
Q
  1. A 30-year-old female patient with a history of infertility is referred for an HSG. She has a past history of pelvic inflammatory disease. HSG reveals multiple small outpouchings from the uterine cavity. What is the diagnosis?

A. Salpingitis isthmica nodosa.

B. Asherman syndrome.

C. Adenomyosis.

D. Endometritis.

E. Multiple endometrial polyps.

A
  1. C. Adenomyosis.

This is a condition in which the endometrium extends into the myometrium in either a diffuse or a focal distribution. It generally manifests as pelvic pain or abnormal bleeding. It is more commonly detected on MR imaging as thickening of the junctional zone or on ultrasound as diffuse or focal heterogenous myometrium. On HSG, adenomyosis appears as small diverticula extending from the endometrial cavity into the myometrium. Salpingitis isthmica nodosa appears as small outpouchings from the isthmic portion of the fallopian tube. Multiple uterine synechiae (linear filling defects in the uterine cavity) associated with infertility is known as Asherman syndrome. Polyps would appear as filling defects on HSG.

94
Q
  1. A 26-year-old female patient is referred to the gynaecology team with a history of primary infertility and oligomenorrhoea. On examination she is hirsute and has a body mass index (BMI) of 31. Which of the following MRI findings are most consistent with her diagnosis?

A. High T2WI signal C-shaped cystic masses with thin, longitudinally oriented folds along their interior in both adnexae.

B. Multiple small peripheral rounded high T2WI signal areas with hypointense central stroma in both ovaries.

C. Normal ovaries.

D. A small ‘banana’-shaped uterus with a single fallopian tube.

E. A junctional zone of 14 mm with multiple hyperintense T2WI foci.

A
  1. B. Multiple small peripheral rounded high T2WI signal areas with hypointense central stroma in both ovaries.

The clinical features suggest polycystic ovarian syndrome (PCOS). The diagnosis is based on hormonal imbalance and patients often show an abnormality in the ratio of luteinizing hormone to follicle stimulating hormone. The MRI findings are those described, with multiple small peripheral cysts with low signal central stroma. MRI is not specific or sensitive, and 25% of patients with PCOS can have normal appearing ovaries. Multiple small, T2WI hyperintense cysts have been seen in patients with anovulation, medication-stimulated ovulation, or vaginal agenesis. The other descriptions are associated with infertility, but not particularly hirsutism or obesity. Option A describes bilateral hydrosalpinx. Option D describes a unicornuate uterus, which is associated with the poorest foetal survival among all uterine anomalies. Option E describes adenomyosis and using a junctional zone thickness of 12 mm or above optimizes the accuracy of MRI for this diagnosis.

95
Q
  1. A 43-year-old woman with a history of breast carcinoma undergoes a CT of abdomen for abdominal pain and menorrhagia. This reveals an enlarged uterus and she proceeds to MRI. The normal T2WI zonal anatomy of the uterus is preserved. The endometrial stripe is of high T2WI signal and measures 14 mm in diameter, and the myometrium is thickened. Lattice-like enhancement of the high-signal T2WI endometrial area is demonstrated on T1WI post contrast administration. There is no evidence of myometrial invasion. What is the diagnosis most consistent with these findings?

A. Intrauterine contraceptive device (IUCD).

B. Tamoxifen therapy.

C. Lymphoma of the uterus.

D. Endometrial stromal sarcoma.

E. Pelvic congestion syndrome.

A
  1. B. Tamoxifen therapy.

The normal endometrial stripe is of high T2WI signal and measures 3–6 mm in diameter in the follicular phase and 5–13 mm in the secretory phase. The description of the endometrium in this case is consistent with endometrial hyperplasia, but there is in addition myometrial enlargement. An enlarged uterus is frequently encountered in the presence of endogenous or exogenous hormonal abnormalities. In these cases the uterus usually has normal zonal anatomy, although the signal intensity of the endometrium and myometrium is abnormally increased. However, with tamoxifen, the uterus can display marked zonal anatomy distortion. It is a weak oestrogen agonist and can result in endometrial hyperplasia, polyps, and carcinoma. The findings of multiple cysts or lattice-like enhancement of the endometrium post contrast are encountered frequently in relation to tamoxifen therapy and favour a benign diagnosis. IUCDs are widely used for contraception. A study has found that the IUCD-bearing uterus is enlarged. IUCD placement most likely results in myometrial hypertrophy. The associated findings are symmetrical globular enlargement of the uterine corpus, cervical elongation and enlargement, and diffuse or localized myometrial thickening. The IUCD will usually be seen as a band of low signal intensity within the endometrium. The uterus is rarely the primary site for lymphoma, but when it does occur, the most common manifestation is diffuse symmetrical uterine enlargement with relatively high signal on T2WI and epithelial preservation: the endometrium is usually normal. The myometrium will also lack its standard zonal appearance. There will almost always be associated lymphadenopathy. Endometrial stromal sarcoma invariably exhibits myometrial involvement. Imaging will typically show a large mass that replaces the endometrial cavity and infiltrates the myometrium: at the very least the endometrial-myometrial border will be obscured. Myometrial involvement is commonly so extensive that a myometrial component predominates. Bands of low signal may be present within the area of myometrial invasion and these correspond to preserved bundles of myometrium at histologic examination. Extension along the vessels, fallopian tubes, or ligaments is another characteristic of the tumour. Pelvic congestion syndrome involves symptoms, including chronic pelvic pain, that are associated with dilated ovarian veins and pelvic varices resulting from left renal vein reflux. This results in a thickened myometrium containing multiple large signal voids, the latter corresponding to engorged arcuate vessels. Additional findings include varicose veins around the uterus and ovaries with retrograde filling of ovarian veins.

96
Q

(Ped) 68. A 15-year-old female undergoes an ultrasound of pelvis that demonstrates uterine abnormality with a differential diagnosis of bicornuate or septate uterus. An MRI of the pelvis is requested for further assessment. Which of the following findings on MRI is suggestive of a bicornuate uterus?

A. Two uterine cavities.

B. Fundal concavity of less than 1 cm.

C. Intercornual distance of more than 4 cm.

D. Convex external fundal contour.

E. Thin fibrous low-intensity septum separating the uterine cavities.

A
  1. C. Intercornual distance of more than 4 cm.

Incomplete fusion of the Mullerian ducts results in bicornuate uterus. MRI demonstrates divergent uterine horns with an external fundal cleft that is more than 1 cm deep and an intercornual distance of more than 4 cm. In bicornuate bicollis uterus, two separate cervical canals are seen. This is distinguished from uterus didelphys by a greater degree of fusion between the horns in the lower uterine segment. Septate uterus results from incomplete septal resorption following Mullerian duct fusion. It is the most common uterine anomaly (55%). At MRI, the external fundal countour may be convex, flat, or minimally concave (less than 1 cm deep). MRI is also useful in characterizing the type (fibrous/ muscular) and extent of the septum. Differentiation between bicornuate and septate uterus is important, as septate uterus is amenable to surgical management.

97
Q
  1. A 14-year-old female presents with a history of cyclic pelvic pain. Speculum vaginal examination reveals a bulging vaginal mass. An MRI of the pelvis demonstrates divergent uterine horns with a deep midline fundal cleft, two separate uterine cavities, two separate cervices, and a unilateral hemivaginal septum causing hematometrocolpos. There is associated renal agenesis on the side of the hemivaginal septum. What is the primary uterine anomaly?

A. Uterus didelphys.

B. Uterine bicornuate bicollis.

C. Septate uterus.

D. Arcuate uterus.

E. Imperforate hymen

A
  1. A. Uterus didelphys.

This is caused by complete failure of fusion of the paramesonephric ducts, resulting in a completely duplicated system (two uterine cavities and two cervices) with no communication between the two cavities. It is associated with complete or partial vaginal septum in 75% of cases, which can result in obstruction and haematometrocolpos. Ipsilateral renal agenesis is associated with a vaginal septum. Bicornuate uterus is caused by partial fusion of the paramesonephric ducts. Bicornuate bicollis uterus demonstrates a septum extending from the fundus to the external os, but some degree of communication remains between the two horns. Septate uterus is the most common mullerian/paramesonephric duct anomaly. It is also the most common anomaly associated with infertility and recurrent miscarriage. A complete or partial septum divides the uterine cavity, but there is no fundal cleft. Arcuate uterus is associated with a small indentation in the fundal endometrial canal but the external contour is normal. It has no pathological consequence and is considered by some to be a normal variant.

98
Q

(Ped) 73. A 14-year-old girl presents with sudden onset of severe right-sided lower abdominal pain, nausea, and vomiting. An ultrasound of the pelvis is requested with a suspected diagnosis of ovarian torsion. Which of the following is the most constant ultrasound finding in ovarian torsion?

A. Enlarged ovary.

B. Absent ovarian blood flow.

C. Pelvic free fluid.

D. Twisted ovarian pedicle.

E. ‘String of pearls’ sign.

A
  1. A. Enlarged ovary.

Ovarian torsion can occur in all age groups, with the highest prevalence in the reproductive age group. Large heavy cysts and cystic neoplasms commonly predispose to ovarian torsion. Torsion of normal ovaries is unusual but more common in adolescents. Clinical symptoms are often nonspecific c and therefore imaging is routinely requested. On ultrasound, an enlarged ovary is the most constant finding. The enlarged ovary may be heterogenous due to haemorrhage and oedema. Other findings include multiple small cysts aligned in the periphery of the enlarged ovary (string of pearls sign), coexistent mass, pelvic free fluid, and twisted ovarian pedicle. Benign cystic teratoma is the most common tumour predisposed to ovarian torsion. Absent arterial flow is described as the classic colour Doppler finding. However, the most frequent Doppler finding is decreased or absent venous flow with or without reduced arterial flow. A twisted ovarian pedicle with a ‘ whirlpool’ sign is a useful finding on colour Doppler.

99
Q
  1. A 37-year-old woman presents with a watery vaginal discharge and attends for an MRI of pelvis. She becomes quite claustrophobic at the end of the scan and you are called to assess her as she has been hyperventilating and the radiographers have become concerned. As you reassure her, you notice some peri-oral pigmentation. The MRI reveals a multicystic lesion (high T2WI and low T1WI signal) in the uterine cervix with a solid (low signal T1WI and T2WI) component in the deep cervical stroma. You note from the picture archiving and communication system (PACS) system that a barium enema previously revealed several colonic polyps. What is the likely cause for the MRI findings?

A. Malignant melanoma of the cervix.

B. Carcinoid tumour of the cervix.

C. Cervical pregnancy.

D. Minimal deviation adenocarcinoma of the cervix.

E. Invasive cervical squamous cell carcinoma.

A
  1. D. Minimal deviation adenocarcinoma of the cervix.

This is also known as adenoma malignum and, as in this scenario, is often associated with Peutz– Jeghers syndrome (characterized by mucocutaneous pigmentation, multiple hamartomatous polyps of the GI tract, and mucinous tumours of the ovary). Adenoma malignum makes up about 3% of adenocarcinoma of the cervix. Its MRI appearances are as described in the question, but the differential diagnosis includes deep nabothian cysts, florid endocervical hyperplasia, and even well-differentiated adenocarcinoma. It disseminates into the peritoneal cavity even in the early stage of the disease and its response to radiation or chemotherapy is poor. Cervical squamous carcinoma makes up to 90% of cervical carcinoma. The tumour is of high signal compared to the hypointense cervical stroma, but not cystic as in our vignette. It advances predominantly by direct extension and local spread; haematogenous dissemination is only occasionally seen in the form of hepatic metastases. Carcinoid tumour of the cervix is a subgroup of small cell carcinoma of the cervix. It cannot be differentiated from squamous cell carcinoma of the cervix on MRI findings. Malignant melanoma of the female genital tract accounts for 1–5% of all melanomas. It usually occurs in the vaginal mucosa and occasionally involves the cervix. Malignant melanoma arising in the cervix is very rare (only about 30 reported cases). There is usually high signal intensity on T1WI. The incidence of cervical pregnancy has been increasing, possibly due to the increased number of induced abortions. Reported risk factors include multiparity, prior cervical surgical manipulation, cervical or uterine leiomyomas, atrophic endometrium, and septate uterus. The major symptom is painless vaginal bleeding. At MR it is characterized by a mass with heterogenous signal intensity and a partial or complete dark ring on T2WI sequences. As it contains haematoma, it often consists of some high signal on T1WI.

100
Q
  1. A 28-year-old para 0 + 0 female patient is referred from the gynaecology team for an MRI of pelvis, after presenting with pain in the RIF. On TVUS they have identified enlargement of the right ovary and have raised the possibility of a mass. On MRI, a 2-cm ovoid lesion is demonstrated within the right ovary. It is of high signal on T1WI and T1WI fat saturation sequences, and low signal on T2WI sequences. There is no evidence of a mural nodule or ascites and the uterus is not enlarged. What is the most likely diagnosis?

A. Fibrothecoma.

B. Endometriotic cyst.

C. Brenner tumour.

D. Simple follicular cyst.

E. Endometroid carcinoma.

A
  1. B. Endometriotic cyst.

Also known as a chocolate cyst, the methaemoglobin within the lesion causes T1 shortening, resulting in increased signal on the T1WI sequence. This high signal remains and becomes more conspicuous on the T1WI with fat saturation sequence. Endometriosis is characterized by the presence of tissue resembling endometrium outside the uterus. The ovaries are the most commonly involved site, and endometriotic cysts usually have a thick fibrotic wall with chocolate-coloured hemorrhagic material. An endometriotic cyst may be high or low signal on T2WI sequences. Chronic cyclical haemorrhage and increased viscosity of the cyst material will produce T2 shortening, leading to the low signal, or ‘T2 shading’. The patient usually presents with cyclical pain. The cysts have a propensity for multicentric growthand are often associated with fibrous adhesions. These latter features increase the MR sensitivity. Small peritoneal implants of endometriosis may be identified elsewhere and their conspicuity is increased by the use of fat saturation techniques. Endometroid and clear cell carcinoma of the ovary are malignancies associated with endometriosis and these represent 17.5 and 7.4% of ovarian carcinomas, respectively. Chocolate cysts with multiple locules, mural foci, or nodules within the cyst are suspicious for malignancy and contrast should be administered if this appearance is seen. The findings in this question are not consistent with a follicular cyst, which would show low T1WI and high T2WI signal. Brenner tumours show low T2WI signal due to their abundant fibrous content and calcification, but also low T1WI signal. Fibrothecomas show intermediate T1WI signal and usually low (although sometimes it can be mixed high and low) T2WI signal. They sometimes show associated uterine enlargement, as they may be oestrogenic.

101
Q

25 A 60-year-old female was admitted with progressive abdominal distension and vague abdominal pain. She is found to have a 20-cm complex ovarian mass on ultrasound and a raised CA-125level. What is the next step in management and staging?

a PET scan

b CT followed by biopsy

C Biopsy of the ovarian mass followed by MRI examination

d Primary surgery

e MRI examination followed by biopsy

A

25 Answer B: CT followed by biopsy

In a patient with a clinical history, ultrasound findings and CA-125 level consistent with ovarian carcinoma, a staging CT examination should be the next investigation. As well as staging the patient this will also need a guided biopsy. Omental biopsy can be undertaken percutaneously via ultrasound or CT guidance but ovarian disease biopsy would usually have to be performed surgically. MRI of the pelvis is used in the investigation of an adnexal mass which is not definitely ovarian in origin as a problem-solving tool.

102
Q

26 A 45-year-old female with a complex ovarian mass and raised CA-125 level underwent staging CT examination. What are the findings consistent with her having stage III disease?

a Both ovaries involved with malignant ascites

b Extension to the fallopian tubes without malignant ascites

C Malignant ascites with implants on the uterus

d Peritoneal metastases

e Liver metastases

A

26 Answer D: Peritoneal metastases

FIGO Staging of Ovarian Carcinoma Answer

(a): FIGO stage IC Answer

(b): FIGO stage IIA Answer

(c): FIGO stage IIC Answer

(e): FIGO stage IV

103
Q

27 A 30-year-old female patient with pelvic pain and irregular periods underwent an ultrasound examination. In the left ovary there was a 4-cm diffuse homogeneous hypoechoic focal lesion with low-level internal echoes. What is the most likely diagnosis?

a Endometrioma

b Functional ovarian cyst

C Polycystic ovary (PCOS)

d Ovarian carcinoma

e Corpus luteum

A

27 Answer A: Endometrioma

Although ultrasound appearances can be variable approximately 95 % of endome- triomas demonstrate the above findings. They may be unilocular or multilocular with thick or thin septae. Rarely, they are anechoic, mimicking a functional ovarian cyst.

104
Q

28 A 30-year-old female with menorrhagia underwent ultrasound examination of the pelvis which demonstrated an adnexal lesion. She went on to have an MRI which, on T1-weighted images, demonstrated a low-signal mass related to the right ovary. What is the most likely diagnosis?

a Endometriosis

b Ovarian fibroma

c Dermoid

d Haemorrhagic mass

e Mucinous cystic neoplasm

A

28 Answer B: Ovarian fibroma

Endometriosis is typically high signal on routine and fat-saturated Ti sequences due to methaemoglobin. Similarly, a haemorrhagic mass would be expected to be bright on T1. Dermoids often contain fat which is high signal on routine Ti sequences and mucinous lesions can also be high signal but not as high as the Ti signal of fat and blood

105
Q

29 A 35-year-old patient with a history of endometriosis presents with a number of clinical symptoms. Which of the following symptoms cannot be explained by the diagnosis of endometriosis?

a Lump at site of previous surgical scar

b Shortness of breath and pleuritic pain at onset of menses

C Haematuria

d Pelvic pain and feeling of fullness

e Aching muscles

A

29 Answer E: Aching muscles

Occurrence in surgical scars, particularly after gynaecological surgery, is recognised. Catamenial pneumothorax occurring at the onset of menses as well as haemothorax and lung nodules can occur in the chest. In the bladder endometrial deposits can cause haematuria and often biopsy is required for definitive diagnosis. Malignant transformation is rare (<1 %) and 75% of the malignancies arise from endometriosis of the ovary

106
Q

30 A 50-year-old female with a history of bronchial malignancy underwent a PET-CT examination and was noted to have increased activity in the pelvis. Increased uptake in which organ or area would indicate sinister pathology?

a Uterine fibroids

b Ovaries

C Cervix

d Blood vessels

e Bladder diverticulum

A

30 Answer C: Cervix

Normal FDG activity is seen in brain, myocardium, liver, spleen, bone marrow, GI tract, testes and skeletal muscle. In the pelvis pitfalls of sites of increased uptake include cyclical uptake in the endometrium and ovaries, renal collecting system (ureters and bladder) and the blood pool.

107
Q

31 A 40-year-old, pre-menopausal woman is noted to have a unilocular adnexal cyst measuring 4 cm on transvaginal ultrasound examination. What is the most appropriate management?

a MRI examination after six weeks

b Pelvic ultrasound examination after four weeks

C CA-125 blood test and referral to gynaecology

d Pelvic ultrasound examination after six weeks

e CA-12 5 and CT examination

A

31 Answer D: Pelvic ultrasound examination after six weeks

Functional cysts are common, and benign features on ultrasound include unilocular thin-walled cysts with no solid component. If the cyst is larger than 3 cm, follow-up ultrasound at a different time in the menstrual cycle is recommended to ensure resolution.

108
Q

32 A 50-year-old peri-menopausal female patient presents with dysfunctional bleeding. She is referred to the Gynaecology Outpatient Clinic and to Radiology for an ultrasound examination. Assuming that her symptoms are due to a gynaecological malignancy, what is the most likely site of disease?

a Endometrial

b Ovarian

C Vulval

d Cervical

e Vaginal

A

32 Answer A: Endometrial.

109
Q

34 A 65-year-old patient with post-menopausal bleeding comes to ultrasound for a pelvic ultrasound examination. What risk factor on the clinical details makes the patient more at risk for developing endometrial carcinoma?

a Multiparity

b Early menopause

c Late menarche

d Hypertension

e Diabetes insipidus

A

34 Answer D: Hypertension

Nulliparity is a risk factor as is unopposed oestrogen therapy and obesity.

110
Q

35 A 55-year-old post-menopausal patient is experiencing post-menopausal bleeding. She is not on HRT. She undergoes pelvic ultrasound examination and was noted to have an endometrial thickness of 8 mm. What is the next appropriate step in management?

a Discharge patient with no follow-up

b Follow-up ultrasound examination in six weeks

c Referral to gynaecologist for Pipelle biopsy or hysteroscopy

d MRI examination for endometrial staging and referral to gynaecology

e CT examination for endometrial staging and referral to gynaecology

A

35 Answer C: Referral to gynaecologist for Pipelle biopsy or hysteroscopy

Measurement of endometrial thickness post menopause: in a patient not on HRT <5 mm homogeneously echogenic, in a patient on HRT <8 mm. In premenopausal patients: menstrual phase (1-4mm), proliferative phase (5-7mm), periovulatory phase (up to 11 mm) and secretory phase (up to 16 mm). There are alternative diagnoses that increase endometrial thickness such as endometrial polyps and endometrial hyperplasia and therefore radiological staging is performed once there is a histological diagnosis.

111
Q

36 A 60-year-old female patient presented with post-menopausal bleeding and was found to have endometrial carcinoma. An MRI of her pelvis is performed for staging of the endometrial carcinoma. What signal would you expect normal myometrium, junctional zone and endometrium to return on T2 -weighted images

A

36 Answer C

On T2-weighted images three distinct layers can be seen: the endometrium is high signal, junctional zone is low signal and myometrium medium signal. On T1-weighted images the uterus is medium to low signal intensity

112
Q

37 A 75-year-old lady who had grade 2 endometrial adenocarcinoma diagnosed on endometrial sampling undergoes an MRI examination for staging. If there is cervical invasion but no further spread, what would be the correct staging?

a IC

b II

c IB

d IIIB

e IV

A

37 Answer B: II

FIGO staging of endometrial carcinoma is stage II for cervical invasion irrelevant of the depth of myometrial invasion.

113
Q

24 A 26-year-old patient and her partner are being investigated for infertility. His semen analysis is adequate and she has been proven to be ovulating. She undergoes a hysterosalpingogram which demonstrates a uterine abnormality. What is the most likely diagnosis?

a Bicornuate uterus

b Unicornuate uterus

C Septate uterus

d Uterus didelphys

e Arcuate uterus

A

24 Answer C: Septate uterus

Uterine anomalies are due to failure of fusion of Mullerian duct. Arcuate uterus is the most common uterine anomaly not associated with reproductive failure. Septate uterus is associated with 90% abortion rate which can be treated with excision of the septum.

114
Q

25 A 33-year-old asymptomatic patient has been found to have cervical carcinoma and is undergoing a staging MRI examination. What finding on MRI is most reliable in excluding parametrial invasion?

a Absence of abnormal pelvic lymphadenopathy

b No hydronephrosis or hydroureter on either side

c Tumour volume of less than 90 cu cm

d Hypointense rim of cervical stroma

e Fine nodularity of parametrial tissue

A

25 Answer D: Hypointense rim of cervical stroma

A continuous hypointense rim representing the cervical ring measuring >3 mm is most reliable in excluding parametrial invasion with a quoted specificity of 96-99%. Nodularity and thickening of the parametrial tissue are signs of frank invasion.

115
Q

27 A 40-year-old patient is undergoing MRI examination for possible adenomyosis. Which finding on the MRI would be inconsistent with this diagnosis?

a junctional zone greater than 12 mm

b High signal intensity spots on T2-weighted imaging in the myometrium

C Distinct hypointense transient bulge in the myometrium on T2 -weighted images

d junctional zone: myometrial thickness ratio >40%

e Persistent focal ill-defined round area of low signal in myometrium in contact with the junctional zone on T2 -weighted images

A

27 Answer C: Distinct hypointense transient bulge in the myometrium on T2 -weighted images

Transient areas of low signal in the myometrium are a recognised pitfall in the diagnosis of adenomyosis and reflect myometrial contraction. These areas can also be mistaken for leiomyomas.

116
Q

28 A 35-year-old female patient with cervical carcinoma undergoes MRI examination for local staging. What radiological finding would be consistent with the patient having stage IIB disease?

a 4 cm tumour confined to the cervix

b Parametrial invasion

C Extension into the vagina except the lower third

d Extension to the lower third vagina

e Pelvic side wall invasion

A

28 Answer B: Parametrial invasion

FIGO staging of cervical carcinoma.

Stage IA - preclinical invasive carcinoma,

stage IB - 1 < 4 cm,

stage IB - 2 >4 cm,

stage IIA - vaginal extension excluding lower third vagina,

stage IIB – parametrial invasion,

IIIA - invasion of lower third vagina,

stage IIIB - pelvic side wall invasion and hydronephrosis,

stage IVA - invasion bladder/rectum,

stage NB - distant organ spread.

117
Q

29 A 60-year-old female patient with pelvic pain and distension has had a 15-cm cystic and solid mass demonstrated on ultrasound examination, which is arising from her right ovary. The solid septations within it measure >5 mm. The CA-125 level is within the normal range. What is the most likely diagnosis?

a Epithelial ovarian carcinoma

b Endometriosis

c Secondary deposit from another malignancy

d Benign cyst

e Fibroma

A

29 Answer A: Epithelial ovarian carcinoma

The radiological findings are consistent with a malignant lesion and CA-125 levels are not raised in 20% of malignant ovarian cancers. CA-125level is raised in 60% for mucinous type, 20% for nonmucinous. In benign processes such as fibroids, pregnancy, endometriosis, liver cirrhosis, pancreatitis and PID raised in 30%. Raised in 1 % of normal individuals

118
Q

30 A 29-year-old female three days post partum is unwell with pelvic pain and fever. On clinical examination she has a palpable rope-like abdominal mass. What is the most likely diagnosis?

a Right ovarian thrombosis

b Left ovarian thrombosis

c Bilateral ovarian vein thrombosis

d Ruptured uterus

e Tubo-ovarian abscess

A

30 Answer A: Right ovarian thrombosis

This is the typical clinical scenario for ovarian vein thrombosis, which is an important differential to consider as it has a mortality of 5%. Eighty per cent occur in right ovarian vein, 14% bilateral and only 6% in left ovarian vein. On CT a tubular structure is seen of low density in the location of the vein.

119
Q

35 A 28-year-old female patient who is under gynaecological follow-up for a cystic ovarian mass is admitted as an emergency with sudden onset of severe lower abdominal pain. A mass is palpable in the right iliac fossa. Abdominal and pelvic ultrasound examination demonstrates an enlarged hypoechoic midline mass in addition to the cystic mass and there is free fluid in the pouch of Douglas. What is the most likely diagnosis?

a Enlargement of her existing cystic mass

b Haemorrhage of the cyst

c Malignant change of the cyst

d Torsion of the ovary

e Rupture of the cyst

A

35 Answer D: Torsion of the ovary

A pre-existing ovarian lesion is present in torsion in >50% of cases. The appearances on ultrasound are of a complex mass and Doppler ultrasound may demonstrate absence of arterial Doppler waveforms, but this should not be relied upon due to the dual blood supply from both ovarian and uterine arteries. Immediate surgery is indicated in these cases.

120
Q

36 A 38-year-old patient has chronic pelvic pain on the right side. She undergoes transabdominal and transvaginal ultrasound examination. This demonstrates a 3 cm cyst on the right ovary. What radiological findings would suggest a diagnosis of haemorrhagic cyst rather than a simple cyst?

a Posterior acoustic enhancement

b A’ground-glass’ pattern

C Doppler flow in cyst wall

d Sharply defined wall <3 mm

e Unilocular

A

36 Answer B: A `ground-glass’ pattern

Haemorrhage into a functional cyst is common and can produce echogenic material within the cyst without Doppler signal.

121
Q

37 A 34-year-old patient has had previous investigations for dysmenorrhoea and menorrhagia and was found to have multiple leiomyomas. What radiological sign would be concerning for sarcomatous transformation?

a High signal intensity on T2-weighted sequences

b Hyperintense rim on T2-weighted sequences

C Rapid change in size

d Enhancement following intravenous contrast medium

e Low signal intensity on T2-weighted sequences

A

37 Answer C: Rapid change in size

Malignant transformation is rare (approximately 0.2%) and is very difficult to interpret given the variable appearances and signal characteristics of benign leiomyomas. A rapid increase in size, irregular margin and loss of outer capsule are concerning. High signal on T2-weighted images is seen in cystic degeneration. Low signal on T2-weighted images is seen in hyaline or calcific degeneration.

122
Q

38 A 19-year-old girl complains of a pelvic mass and pressure symptoms. Transabdominal ultrasound examination reveals an adnexal mass. What additional finding would suggest a diagnosis of mature teratoma from another type of germ cell tumour?

a Fat density on plain film

b Elevated B-HCG

c Calcification seen on ultrasound imaging

d Elevated alpha-fetoprotein

e Complex mass on ultrasound imaging

A

38 Answer A: Fat density on plain film

Mature teratoma (dermoid) is the most common ovarian neoplasm and is benign. It contains mature tissue from all three germ cell layers with 88% of cases containing sebaceous material. Calcifications (tooth) can be seen but are also seen in other germ cell tumours whereas fat on plain film is specific.

123
Q

20 An 11-year-old girl who has not started menstruating yet is admitted to hospital with urinary retention. A urinary catheter is inserted in the Emergency Department which drains 1 litre of urine. The paediatric doctor can feel a palpable pelvic mass and ultrasound examination demonstrates a midline pelvic mass which is hypoechoic with low-level echoes within it. The ovaries have normal ultrasound appearances. What is the likely diagnosis?

a Tubo-ovarian abscess

b Haematocolpos

C Perforated appendix with pelvic collection

d Endometriosis

e Uterine anomaly

A

20 Answer B: Haematocolpos

Imperforate hymen at puberty may cause haematocolpos which distends the vagina with blood causing low-level echoes on ultrasound examination.

124
Q

22 A 27-year-old patient with pelvic pain underwent an MRI examination for further assessment of a discrete lesion on her right ovary. What finding would suggest diagnosis of endometriosis rather than haemorrhagic cyst?

a Increased signal intensity on T1-weighted images

b Increased signal intensity on fat-suppressed T2-weighted images

C Multiple further lesions in the pelvis

d Fluid-debris layer on T2-weighted images

e A thin wall

A

22 Answer C: Multiple further lesions in the pelvis

Multiplicity, multilocularity and adhesions to other organs are features that may distinguish endometriomas. They may also have a hypointense thick wall.

125
Q

23 A 58-year-old female patient with post-menopausal bleeding is referred for an ultrasound examination of the pelvis. This demonstrates a focal thickening of the endometrium. What additional finding would suggest a diagnosis of endometrial polyp rather than primary carcinoma of the endometrium?

a Increased echogenicity in the myometrium

b Doppler waveform with resistive index <0.7

C Location at the uterine fundus

d Irregular poorly defined endometrial-myometrial interface

e Vessel visualised within stalk on colour Doppler

A

23 Answer E: Vessel visualised within stalk on colour Doppler

Seventy-nine per cent of endometrial polyps have the appearance of a welldefined smooth hyperechoic homogeneous intracavitary mass but the diagnosis can be confusing particularly if there is heterogene ty

126
Q

24 A 52-year-old patient who is perimenopausal was referred for investigation of menorrhagia and increased frequency of menstruation. A blood sample showed elevated levels of oestrogen and a pelvic ultrasound examination was arranged. On this examination the endometrium measured 3 mm and there was a multilocular cystic adnexal mass measuring 30 cm with thick irregular septations but no intracystic papillary projections. What is the most likely diagnosis?

a Adult granulosa cell tumour

b Germ cell tumour

C Krukenberg tumour

d Clear cell carcinoma

e Brenner tumour

A

24 Answer A: Adult granulosa cell tumour Adult granulosa cell tumour is the most common oestrogenic tumour

127
Q

25 A 35-year-old female patient with post-coital bleeding has a pelvic MRI scan as part of her workup which demonstrates a focal mass in the cervix. What additional finding would suggest a diagnosis of prolapsed submucosal fibroid rather than cervical carcinoma?

a Hypointensity on T1-weighted images

b Hypointensity on T2-weighted images

C Blurring and widening of the junctional zone

d Early contrast enhancement on fat-saturated T1-weighted images

e Disruption of hypointense vaginal wall

A

25 Answer B: Hypointensity on T2-weighted images

Cervical carcinoma is hyperintense on T2-weighted images but non-degenerated leiomyomas are low signal.

128
Q

27 A 21-year-old Afro-American patient is being investigated for menorrhagia and dysmenorrhoea. Ultrasound examination demonstrated several intramural fibroids and also a well-defined adnexal mass. MRI examination showed this to be attached to the uterus and likely to be a subserosal fibroid. What is the most typical signal characteristic on MRI of a subserosal fibroid?

a High signal on T1-weighted images

b Low signal on T1-weighted images

C Intermediate signal on T2-weighted images

d High signal on T2-weighted images

e Low signal on T2-weighted images

A

27 Answer E: Low signal on T2-weighted images

Fibroids have variable appearances on ultrasound but are commonly hypoechoic and can distort the uterine cavity when large. The typical appearances on MRI examination are well defined areas which are low signal on T2-weighted images and are isointense to myometrium on T1-weighted images. The appearances are variable if there is haemorrhage or cystic degeneration

129
Q

30 A 22-year-old female patient is referred to gynaecology with a palpable mass behind the labia minora. She undergoes MRI examination which demonstrates a small mass which is of high homogeneous signal on T2-weighted images. What is the most likely diagnosis?

a Bartholin’s gland abscess

b Bartholin’s gland squamous cell carcinoma

C Bartholin’s gland cyst

d Bartholin’s gland neuroendocrine carcinoma

e Bartholin’s gland adenocarcinoma

A

30 Answer C: Bartholin’s gland cyst

Bartholin’s glands are located behind the labia minora and their ducts can become blocked. This can cause retention of secretions and is most common in women of reproductive age.

130
Q

31 A 79-year-old female patient who has previously had a malignant melanoma excised on her thigh presents to the gynaecology team with a vulvular mass. She undergoes MRI examination which demonstrates a 6 cm mass. What additional finding would suggest a diagnosis of malignant melanoma rather than carcinoma?

a High signal on T1-weighted images

b High signal on T2-weighted images

C Enhancement following intravenous gadolinium

d Heterogeneity

e Infiltration of surrounding structures

A

31 Answer A: High signal on T1-weighted images

Melanin has a paramagnetic effect causing shortening of Ti relaxation time and therefore is high signal on T1-weighted images and low signal on T2-weighted images.

131
Q

32 A female patient has been referred to gynaecology with pelvic pain and the gynaecology team are concerned about salpingitis. What finding would be most suggestive of tuberculosis rather than another cause?
a Tubal content low signal intensity on T2-weighted images b Bilateral disease c Thickened wall of fallopian tube d Dilated fallopian tube e Haemorrhage within tubes

A

32 Answer B: Bilateral disease

Female genital tuberculosis causes salpingitis in 94% and is mostly bilateral. It can also cause tubo-ovarian abscess with extension into extraperitoneal compartment.

132
Q
  1. A 50-year-old woman presents with intermittent pelvic pain. Ultrasound shows a cystic mass in the adnexa. This contains a hyperechoic nodule which produces marked acoustic shadowing. CT shows a fat containing lesion in the pelvis with a fluid level and a small calcifed nodule. What is the most likely diagnosis?

(a) Ovarian dermoid

(b) Liposarcoma

(c) Tubo-ovarian abscess

(d) Endometrioma

(e) Haemorrhagic ovarian cyst

A
  1. (a) Ovarian dermoid

This is a germ cell tumour which contains skin and dermal appendages. These are diagnosed by characteristic ectodermal contents of hair, teeth fat and bone. Identifcation of a sebum-fluid level and calcifcation is diagnostic. There is risk of malignant degeneration, torsion and rupture.

133
Q
  1. A 40-year-old mother of two presents with a right lower abdominal lump near a surgical scar and with a cyclical history of pain. Ultrasound shows a 2 cm solid hypoechoic lesion in the subcutaneous tissue. Doppler shows internal vascularity. The most likely diagnosis is?

(a) Desmoid tumour

(b) Endometriosis

(c) Metastasis

(d) Lymph node

(e) Suture granuloma

A
  1. (b) Endometriosis Endometriosis can be found in surgical scars or needle tracts. Most cases of subcutaneous endometriosis occur in Pfannenstiel incisions. Abdominal wall endometriosis is thought to occur in up to 1% of cases. Clinically it presents as a cyclical painful lump and c n arise many years after surgery.
134
Q
  1. A 31-year-old woman presents with dysfunctional uterine bleeding. Transvaginal ultrasound shows a hypoechoic vascular mass in the cervix. The mass bulges into the endocervical canal and parametrium. On MRI, there is a cervical mass lesion which returns high signal on T2 and poorly defined margins beyond the cervical stroma. The most likely diagnosis is?

(a) Endometrial carcinoma

(b) Cervical carcinoma

(c) Focal adenomyosis

(d) Cervical lymphoma

(e) Prolapsed submucosal fbroid

A
  1. (b) Carcinoma of the cervix

Cervical carcinoma typically presents with bleeding and pelvic pain. MRI is the imaging of choice which shows high signal on T2 images

135
Q
  1. A 35-year-old Asian woman presents with lower abdominal pain and fever. Transvaginal ultrasound shows bilateral, homogenous, extraovarian, tubular lesions containing fluid with featureless walls. What is the most likely diagnosis?

(a) Cystic ovarian tumour

(b) Chocolate cysts

(c) Bilateral hydrosalpinx

(d) Paraovarian cysts.

(e) Small bowel

A
  1. (c) Bilateral hydrosalpinx

This is usually a result of continuous secretion of the tubal epithelium into the lumen of a fallopian tube obstructed at two sides. Ultrasound shows undulating or folded tubular structures which are extraovarian. This may be secondary to endometriosis, adhesions, infection, tubal surgery or ectopic pregnancies

136
Q
  1. A 23-year-old woman presents with recurrent cyclical lower abdominal pain. Ultrasound shows a 4 cm heterogenous cystic mass in the pelvis related to the left ovary. On MRI, the lesion has predominantly high signal on T1, T2 and STIR sequences. What is the most likely diagnosis?

(a) Metastasis

(b) Krukenberg tumour

(c) Ovarian dermoid

(d) Endometrioma

(e) Ectopic pregnancy

A
  1. (d) Endometrioma

MRI is highly sensitive and specifc in the diagnosis of endometrioma. The endometrioma returns high signal on T1 and T2 and STIR sequences due to blood products. On ultrasound the lesion may show diffuse homogenous low-level internal echoes (haemorrhagic debris). Other features may include septations or echogenic material suggesting a clot.

137
Q
  1. A 25-year-old woman with a history of pelvic pain undergoes a transvaginal ultrasound examination. The endometrium is 15 mm thick. Which phase of the menstrual cycle is the patient in?

(a) Proliferative phase

(b) Day 7 after menstruation

(c) Follicular phase

(d) Luteal phase

(e) Day 15 of the cycle

A
  1. (d) Luteal phase

Immediately after menstruation, the endometrium is 1–4 mm thick. In the proliferative phase the endometrium increases to 7–10 mm. It measures 8–12 mm in follicular phase and in the luteal phase it becomes echogenic throughout with a thickness of 8–16 mm.

138
Q
  1. A 30-year-old woman under investigation for pelvic pain has an MRI scan which shows two uterine horns with two completely separate cervices. There is a septum extending into the upper vagina. There is preservation of the endometrial and myometrial widths. What is the most likely diagnosis?

(a) Septate uterus

(b) Bicornuate uterus

(c) Uterus didelphys

(d) Mayer–Rokitansky–Kuster–Hauser syndrome

(e) Arcuate uterus

A
  1. (c) Uterus didelphys

This has complete duplication, with two vaginas, two cervices and two uterine horns. Bicornuate uterus has a fundal cleft with the septum between the two uterine cornua comprised of myometrium. Each uterine horn has a biconvex shape with lateral convex margins. Septate uterus is the most common type of uterine anomaly with a fbrous septum dividing the uterine cavity and the endocervical canal. The fundal contour is convex or flat. Uterine arcuatus show no division of the uterine horns and have a single uterine canal with a saddle shaped fundus on hysterosalpingography.

139
Q
  1. A 65-year-old diabetic woman presents with bleeding per vagina. Ultrasound shows echogenic and irregular endometrium measuring 12 mm in thickness. What is the most likely diagnosis?

(a) Submucosal fbroid

(b) Endometrial polyp

(c) Endometrial carcinoma

(d) Endometrial sarcoma

(e) Uterine sarcoma

A
  1. (c) Endometrial carcinoma

Endometrial carcinoma is the fourth most common female cancer, with a peak between 55 and 65 years. In postmenopausal women endometrial thickness more than 5 mm should be investigated for endometrial carcinoma. Sarcomas are rare in this age group.

140
Q
  1. A 38-year-old woman presents with painless bleeding per-vagina. MRI shows a 2 cm lesion in the cervix which is isointense on T1, hyperintense compared to cervical stroma on T2 and shows contrast enhancement. What is the most likely diagnosis?

(a) Prolapsed submucosal fbroid

(b) Cervical fbroid

(c) Cervical carcinoma

(d) Nabothian cyst

(e) Endometrial polyp

A
  1. (c) Carcinoma of the cervix

Cervical carcinoma is high signal as compared to fbrous cervical stroma on T2 and enhances with contrast.

141
Q
  1. A 60-year-old woman presents with progressive abdominal swelling. Ultrasound shows a large loculated cystic lesion in the lower abdomen, arising from the pelvis. Transvaginal scan shows a right side complex mass in the adnexa with cystic and solid components. The solid components show blood flow with a low-resistive index. What is the most likely diagnosis?

(a) Ovarian dermoid

(b) Endometrioma

(c) Ovarian carcinoma

(d) Tubo-ovarian abscess

(e) Mature teratoma

A
  1. (c) Ovarian carcinoma

Malignant ovarian tumours commonly have solid and cystic components. The solid component has neovascularisation demonstrating a characteristic waveform with a low resistive index.

142
Q
  1. A 36-year-old woman with history of previous miscarriage treated by evacuation of retained products of conception, presents with amenorrhea. Hysterosalpingography shows multiple, irregular, constant flling defects in the uterine cavity which cannot be obscured by contrast flling into the uterine cavity. What is the most likely diagnosis?

(a) Adenomyosis

(b) Submucosal fbroids in uterus

(c) Polyps

(d) Asherman’s syndrome

(e) Subserosal uterine fbroids

A
  1. (d) Asherman’s syndrome

Synechiae or intrauterine adhesions were described by Asherman and are usually a result of uterine curettage or evacuation of retained products of conception. The hysterosalpingogram fndings are diagnostic

143
Q
  1. A 25-year-old woman presents with dysmenorrhoea. MRI of the pelvis shows that the junctional zone of the uterus measures 15 mm and returns low signal on T2 images. On T1 images, there are multiple high signal intensity foci in the junctional zone.What is the most likely diagnosis?

(a) Normal for age

(b) Adenomyosis

(c) Endometrial carcinoma

(d) Uterine infection

(e) Gestational trophoblastic disease

A
  1. (b) Adenomyosis

This is the presence of endometrial tissue within the myometrium with secondary smooth muscle hypertrophy/hyperplasia. MRI features are typically as described in the fndings. Foci of high signal on T1 represent endometrial rests and/or haemorrhages.

144
Q
  1. A 37-year-old woman was diagnosed with cervical carcinoma. MRI scan demonstrates a 4 cm tumour invading the upper third of the vagina and infiltrating the left parametrium. No other organ involvement is seen. What is the most accurate TNM staging for this tumour?

(a) T1b1

(b) T2

(c) T2b

(d) T3b

(e) T4

A
  1. (c) Stage T2b

Stage 1 lesions are confned to cervix. Stage 2a lesions involve the vagina, while T2b tumours invade the parametrium as well. Stage 3 lesions involve the lower third of vagina and/or the lateral pelvic wall. Stage 4 tumours are seen to invade other surrounding organs such as the bladder and rectum.

145
Q
  1. A 35-year-old obese woman with history of irregular periods and hirsutism presents for ultrasound examination. A transvaginal ultrasound demonstrates bilateral enlarged ovaries with multiple hypoechoic cysts, ringed in the periphery of the ovaries measuring 5–10 mm in size. What is the most likely diagnosis?

(a) Endometriosis

(b) Stein–Leventhal syndrome

(c) Ovarian dermoids

(d) Tubo-ovarian abscesses

(e) Brenner tumours

A
  1. (b) Stein–Leventhal syndrome

Also known as polycystic ovary disease, patients may have reduced fertility, hirsutism, obesity and menstrual irregularities. The ovaries are generally enlarged with multiple small follicles measuring less than 10 mm, usually subcapsular

146
Q
  1. A 36-year-old woman with primary infertility was sent for hysterosalpingography. Which of the following is an absolute contraindication to this procedure?

(a) Previous caesarean section

(b) Reconstructive tubal surgery in last 6 months

(c) Menstruation (d) Congenital abnormalities of the genitalia

(e) Treated pelvic infection

A
  1. (c) Menstruation

Ongoing bleeding at the time of examination is an absolute contraindication to hysterosalpingography. It increases the risk of infection, and risks flushing endometrial tissue into the abdomen. Recent tubal surgery within last 6 weeks is also a contraindication for this procedure. Other contraindications include pregnancy, immediate pre- and post-menstrual phases, recent untreated pelvic infection and contrast allergy.

147
Q
  1. A 12-year-old pre-pubertal girl presents with vaginal bleeding. What is the most likely diagnosis?

(a) Vaginal foreign body

(b) Endometrial hyperplasia

(c) Ovarian fbroma

(d) Sarcoma

(e) Endometrioma

A
  1. (a) Vaginal foreign body

This is a very common cause of vaginal bleeding in pre-pubertal girls. Other causes include vaginal rhabdomyosarcoma, precocious puberty, haemangioma and vascular malformation.

148
Q
  1. A 37-year-old woman presents with menstrual irregularities. Ultrasound shows a right adnexal abnormality. MRI shows a 3 cm well-defned lesion in the right adnexa which returns high signal on T1. What is the most likely diagnosis?

(a) Fibroma

(b) Brenner tumour

(c) Ovarian dermoid

(d) Pedunculated leiomyoma

(e) Fibrothecoma

A
  1. (c) Ovarian dermoid

Fibrous lesions in the adnexa are of low signal intensity on MRI. Ovarian dermoids return high signal because of their fat content, with signal drop-out on fat suppression images. Other causes of lesions which may return high signal on T1 include endometrioma, mucinous cystic neoplasm, haemorrhagic cysts and ovarian carcinoma.

149
Q
  1. Regarding uterine leiomyomas, which of the following are correct? (T/F)

(a) Simple leiomyomas can metastasise.

(b) Cystic degeneration occurs in 30% of cases.

(c) Red degeneration is associated with the contraceptive pill.

(d) Calcification is usually peripheral.

(e) They are typically of low T2 signal relative to the surrounding myometrium.

A

Answers:

(a) Correct

(b) Not correct

(c) Correct

(d) Not correct

(e) Correct

Explanation:

In leiomyomas (fibroids) cystic degeneration is rare, seen only in 4 % cases. Calcification is seen associated with red degeneration, and is typically scattered and amorphous marking the site of hyaline degeneration.

150
Q
  1. Which of the following statements are correct? (T/F)

(a) The risk of an ectopic pregnancy is higher in patients with a unicornuate uterus.

(b) Nabothian cysts occur in the posterolateral wall of the lower third of the vagina.

(c) The uterus is derived from the paired mullerian ducts.

(d) A unicornuate uterus is rarely associated with other anomalies.

(e) Gartner’s duct cysts are typically located anterolateral to the upper two thirds of the vagina.

A

Answers:

(a) Correct

(b) Not correct

(c) Correct

(d) Not correct

(e) Correct

Explanation:

Bartholin’s cyst occurs in posterolateral wall of lower third of vagina. Mucous retention within endocervical glands is known as nabothian cyst and can be seen in any wall of cervix. 40% cases of unicornuate uterus is associated with renal and ureteric anomalies

151
Q
  1. Regarding polycystic ovaries, which of the following are correct? (T/F)

(a) Cysts of 5-8mm are characteristically present throughout the ovary.

(b) They are seen in patients with trophoblastic disease.

(c) There is an increased risk of endometrial carcinoma.

(d) The ovaries are enlarged on ultrasonography in 95% of cases.

(e) There is a decrease in the ratio of luteinising hormone to follicular stimulating hormone.

A

Answers:

(a) Not correct.

(b) Not correct.

(c) Correct.

(d) Not correct

(e) Not correct.

Explanations:

The cysts are typically subcapsular in polycystic ovaries with central stroma showing increased echogenicity. In 25% cases ovaries are hypoechoic with no demonstrable follicle. The ovaries are bilaterally enlarged (more than 14 cm3) in 70% cases and of normal size in 30% cases. There is increase in ratio of luteinising hormone to follicular stimulating hormone resulting in immature follicles. Trophoblastic disease typically causes hyperstimulation of ovaries with multiseptated cysts secondary to increased HCG levels.

152
Q

@#e 28. Which of the following are correct regarding ovarian cancer: (T/F)

(a) It is the commonest gynaecological malignancy.

(b) It is associated with colorectal cancer.

(c) CA-125 is specific for ovarian cancer.

(d) CT only has a pre-operative staging accuracy of 50%.

(e) Doppler ultrasound may help with differentiating benign from malignant disease.

A

Answers:

(a) Not correct

(b) Correct

(c) Not correct

(d) Not correct

(e) Correct

Explanation:

Endometrial cancer is the most common gynaecological malignancy. CA-125 is not specific for ovarian cancer, it is increased in benign conditions like fibroids, endometriosis and inflammatory pelvic disease. CT only has a pre-operative staging accuracy of 70% - 90%.

153
Q
  1. Which of the following are correct regarding concerning ovarian teratomas: (T/F)

(a) Mature teratomas are usually multi-loculated.

(b) They are a known cause of peritonitis.

(c) Fat attenuation on CT is diagnostic for mature cystic teratomas.

(d) Sebaceous fluid has low signal on T1 weighted MR.

(e) Calcification indicates a malignant teratomas.

A

Answers:

(a) Not correct.

(b) Correct.

(c) Correct

(d) Not correct

(e) Not correct

Explanation:

Mature Teratomas are unilocular. Sebaceous fluid has high signal on T1WI. Calcification outside the mural nodule is suspicious of malignancy.

154
Q
  1. Which of the following are correct regarding mullerian duct abnormalities: (T/F)

(a) The upper two thirds of the vagina arise from the mullerian ducts.

(b) Affect 1-5% of women of reproductive age.

(c) Uterus didelphys is the most common anomaly.

(d) Septate uterus is associated with infertility.

(e) Are associated with renal agenesis.

A

Answers:

(a) Correct

(b) Not correct

(c) Correct

(d) Correct

(e) Correct

Explanation:

0.1% to 0.5% women of reproductive age are affected by congenital abnormalities of uterus. Approximately 25% of patients with anomaly have sub-fertility

155
Q
  1. Which of the following are correct regarding characteristic MRI features of adenomyosis: (T/F)

(a) Foci of high signal within the myometrium on T1 weighted MRI.

(b) Focal thickening of the junctional zone.

(c) Foci of high signal within the myometrium on T2 weighted MRI.

(d) Significant displacement of the endometrial cavity.

(e) Large feeding vessels.

A

Answers:

(a) Correct

(b) Correct

(c) Correct

(d) Not correct

(e) Not correct

Explanation:

On MRI adenomyosis and leiomyoma have overlapping features. Features favouring adenomyosis are: 1) poorly defined borders 2) oval shaped lesion along endometrium 3) minimal mass effect on endometrium relative to size of lesion 4) linear striations perpendicular to endometrium radiating into myometrium 5) absence of large feeding vessels.

156
Q

(GIT) 38. In recurrent cervical cancer, which of the following are correct? (T/F)

(a) Hydronephrosis occurs in 70% of cases.

(b) Para-aortic nodes are generally involved before pelvic side-wall nodes.

(c) Liver metastases occur in 30% of cases.

(d) The rectum is rarely involved.

(e) Adrenal gland involvement is rare.

A

Answers:

(a) Correct

(b) Not correct

(c) Correct

(d) Not correct

(e) Not correct

Explanation:

The paracervical, parametrial, obturator and iliac nodes are involved first. Later there is spread to the common iliac and para-aortic nodes with worse prognosis. The adrenal glands is next common solid organ involvement after liver. The kidneys and pancreas are involved rarely. The recurrent tumour commonly involve the rectum and recto-vaginal fistula may develop.

157
Q

(GIT) 39. Which of the following are correct regarding ovarian cancer? (T/F)

(a) The liver is the most common site of haematogenous metastasis.

(b) Serous cystadenocarcinoma contains calcification in 30% of cases on CT.

(c) Mediastinal lymph node involvement is a rare finding.

(d) About 75% of ovarian neoplasms are benign.

(e) Lesions greater than 4cm on CT are suggestive of malignancy.

A

Answers:

(a) Correct

(b) Correct

(c) Not correct

(d) Correct

(e) Correct

Explanation:

In the ovary cancer, lymph node spread is typically along the path of the gonadal vessels to the para-aortic nodes and along the parameterial channels to the external iliac and hypogastric group.